Сохранен 457
https://2ch.hk/spc/res/338037.html
Домены arhivach.top и arhivach.site временно не функционируют! Используйте домен ARHIVACH.XYZ.
24 декабря Архивач восстановлен после серьёзной аварии. К сожалению, значительная часть сохранённых изображений и видео была потеряна. Подробности случившегося. Мы призываем всех неравнодушных помочь нам с восстановлением утраченного контента!

Тред тупых вопросов №70

 Аноним 04/05/17 Чтв 21:20:47 #1 №338037 
1-jan-hendrik-oort-dutch-astronomer-emilio-segre-visual-arc[...].jpg
91413790719cfca39375o.jpg
14297716991400.jpg
Galaxy Cat D Tumblr We Heart It.jpg
Тред вопросов о жизни, Вселенной и всем таком.

Спрашиваем то, за что в других местах выдают путёвку в биореактор. Здесь анонимные ученые мирового уровня критически рассмотрят любые гениальные идеи и нарисованные в Paint схемы.

Предыдущий тут: https://2ch.hk/spc/res/334598.html
Аноним 04/05/17 Чтв 21:30:22 #2 №338043 
1.Что там в точках лагранжа системы земля-солнце? Точно ничего нет? Кем проверено? Во всех.
2. Пик. Есть 2 ЧД. Если они приблизятся они будут очень быстро вращаться. Но такая орбита нестабильна и в нормальных условиях они слипнутся. Но у нас есть противовес, что не даст им это сделать и они будут вращаться на искусственной орбите.
Я так понимаю, что они начнут испарятся излучая гравитацию. Это так?
Что если так испарять их пока они не станут меньше планковского объёма, пропадёт ли сама ЧД или она будет жить пока снова не наберёт массу.
Аноним 04/05/17 Чтв 21:30:45 #3 №338044 
ffff.png
пик
>>338043
Аноним 04/05/17 Чтв 21:44:02 #4 №338048 
>>338043
>Что там в точках лагранжа системы земля-солнце?
В L4 болтается земной троянец 2010 TK7. Правда не совсем в ней, у него наклонение 20 градусей. В L5 никто ничего не нашел. L1, L2, L3 нестабильны.
>Кем проверено?
Специализированными инструментами по поиску астероидов, в основном в последние лет 10-15.
Аноним 04/05/17 Чтв 21:46:38 #5 №338050 
>>338048
http://www.minorplanetcenter.net/db_search/show_object?object_id=2010+TK7&commit=Show кликаешь Interactive Orbit Sketch и видишь орбиту визуально.
Аноним 04/05/17 Чтв 21:59:15 #6 №338053 
>>338043
>1.Что там в точках лагранжа?
В каких именно?
>Точно ничего нет?
Троянские астероиды есть.
Точки Лагранжа Земля-Солнце используются для парковки космических обсерваторий.

>Но у нас есть противовес
Нету, задача трех тел имеет несколько частных решений, но твое в них не входит.
>Я так понимаю, что они начнут испарятся излучая гравитацию.
Нет, не правильно понимаешь, они потеряют энергию на гравитационное излучение и сольются в одну.
Аноним 04/05/17 Чтв 21:59:17 #7 №338054 
Если тупо взлететь на высоты ГСО то ты будешь вращаться по ней или надо ускорятся в бок? На какой высоте можно не волноваться, что ты ебанёшься если полетишь вверх без угловой скорости?
Аноним 04/05/17 Чтв 22:09:12 #8 №338058 
>>338054
17км/с
Аноним 04/05/17 Чтв 22:24:58 #9 №338062 
кмс.png
>>338058
Я имею ввиду так.
Аноним 04/05/17 Чтв 23:16:48 #10 №338072 
>>338062
круговая орбита с такой орбитальной скоростью находится за л2 и, соответственно - сферой притяжения землюхи, так что либо пиздовать в л2, либо 17 км/с третьей космической либо 550км/с четвёртой
такие дела
Аноним 04/05/17 Чтв 23:52:00 #11 №338078 
>>338072
Обнаружена граница притяжения земли, все в рельсы!
sageАноним 04/05/17 Чтв 23:53:59 #12 №338079 
>>338078
Очевидно имеется в виду https://en.wikipedia.org/wiki/Hill_sphere

другой анон
Аноним 04/05/17 Чтв 23:54:15 #13 №338080 
>>338078
ты ёбанутый?
Аноним 05/05/17 Птн 00:02:22 #14 №338082 
>>338080
Немножка. Но ведь гравитация не выключится при пересечении сферы влияния. Как анон летел так и будет лететь. Даже круговое вращение вокруг земли сохранится.
Кратеры часть 1. Кратеры и шизики. Аноним 05/05/17 Птн 00:09:24 #15 №338083 
SmallCraterDepth2Diam.png
LargeCraterDepth2Diam.png
RatioDepDia.png
Шрётер.png
>>337924 >>337858 >>337905 >>337907 >>337908

Как обещал о кратерах.
Начнем с начала и обратимся к первому посту с вопросом. Напомню, что анон вопрошал: «Парни, а что это за бред форсится про одинаковую глубину лунных кратеров независимо от диаметра? Я не понимаю, это уже высасывают из пальца сенсации или есть какая-то подоплека под этим??»

Да, это бред. Про то, что кратеры на Луне одинаковой глубины говорят всякие маргинальные хуесосы, которые с вопросом не знакомы, и просто перекудахтывают слова зоганутых шизиков. Зоганутые шизики твердят, что Луна – искусственный объект\база инопланетян\база Рейха\родина слонов – вот и вся подоплека.

Конечно, кратеры на Луне очень разные и разной глубины. Например, очень известный кратер Тихо имеет глубину в 4700 м! Прописью: четыре километра семьсот метров. При этом кратер Годдард, не смотря на то, что больше кратера Тихо в диаметре на ~8 километров, имеет глубину всего лишь в 1700 метров. Тогда как кратер Груйтуйзен, имея диаметр жалких 15 км, глубже, чем кратер Годдард - 1870 метров. Кратер Гарольд имеет глубину всего лишь в 360 метров. Можно продолжать и продолжать. Согласитесь, что 4700 и 360 метров – это несколько разная глубина?

Вот картинки, которые показывают распределение ударных кратеров по диаметру и глубине, статистика богатая – более 1900 кртатеров. Можно ознакомиться.
Первая кртинка – до 16 км в диаметре, вторая до 240 км, третья ограничена 100 км. Более того, Луна, как самый доступный для изучения объект, был изучен вдоль и поперек давно. Еще Иоганн Шрётер в своем труде с непроизносимым названием «Selenotopographische Fragmente zur genauern Kenntniss der Mondfläche, ihrer erlittenen Veränderungen und Atmosphäre, sammt den dazu gehörigen Specialcharten und Zeichnungen» конца 18-го века описал глубины и возвышенности да и топографию Луны в целом. Фотографий тогда никто не делал, поэтому каждый астроном был немного художник. Извините за качество изображения, библиотекари из Высшей технической школы Цюриха оцифровали книгу довольно шакально.



Кратеры часть 2. Почему шизики легко наебывают людей. Аноним 05/05/17 Птн 00:09:51 #16 №338084 
Аризонский кратер.jpg
krater7.jpg
wacaristhighsun.jpg
M1096850878LRthumb.png
Теперь про то, откуда в головах шизиков поселились и закрепилась идея об одинаковой глубине кратеров.
Я, конечно, не психиатр, но мне кажется, я знаю разгадку – все дело в том, что кратеры рассматривают обычно сверху. В таком ракурсе, при особенностях зрения человека, они кажутся практически одинаковыми в глубину, даже плоскими, если края достаточно ровные, а цвет кратера ровный. Не будем ходить пока по Луне, сгоняем в США, в Аризону. Вот на пике – знаменитый Аризонский кратер, так, как он выглядит на гугл-мапс. Круглая чаша, глубина не воспринимается, а ведь он очень глубокий, вот фото рядом. Его глубина достигает 230 метров. Это, на секундочку, высота башни "Империя" Москоу-Сити. Или небоскреб Enterprise Plaza в Хьюстоне, если хотите.
Теперь летим на Луну и смотрим, например, кратер Аристарх. Вот так он выглядит сверху: казалось бы не глубокий, но стоит добавить тень, или посмотреть на него с другого ракурса, он сразу приобретает объем и глубину. А глубина не маленькая – больше 3000 метров.
Таким образом, непосвященный человек, даже если немного сомневается угарном бреде шизиков, смотрит на плоские изображения, подсунутые зогонутыми, видит кратеры сверху и наебывается, глаза его обманывают, таким образом в мире становится одним зомби больше.

Спасибо за внимание, в следующем выпуске будет про кратерообразование и про то, почему на видимой стороне Луны кратеры глубже, чем на обратной.
Аноним 05/05/17 Птн 00:13:05 #17 №338085 
>>338082
> гравитация не выключится при пересечении сферы влияния
к чему это вообще?
условие реквеста
> можно не волноваться, что ты ебанёшься если полетишь вверх без угловой скорости
выполняться не будет, как и нарисованная картиночка
Аноним 05/05/17 Птн 06:48:33 #18 №338119 
800.jpg
Безымянный.png
Заметил такую вещь, если смотреть на уличные фонари в расфокусе то на размытой блямбе четко видно мелкое говно которое заслоняет свет от фонаря и которое не увидишь сфокусировавшись и увеличив сам фонарь. Применяется ли подобный метод где-то в астрономии для наблюдения мелкого говна, или мне бежать в нобелевский комитет за открытие? Иллюстрации для пояснения.
Аноним 05/05/17 Птн 09:04:41 #19 №338130 
>>338119
Это говно в твоем глазу/объективе.
Аноним 05/05/17 Птн 09:05:34 #20 №338131 
>>338083
>>338084
От души.
Аноним 05/05/17 Птн 10:30:46 #21 №338136 
>>338119
дефект стекловидного тела
Аноним 05/05/17 Птн 10:51:17 #22 №338138 
>>338136
>>338130
Сука тупые идиоты. Видны ветки и листья и прочие объекты.
Аноним 05/05/17 Птн 10:53:47 #23 №338139 
>>338083
>>338084
Огромное спасибо, прочитал не отрываясь. Я тот самый анон, который задал вопрос изначально, если что. Жду продолжение.
Хочется сказать вот что, если просто вбивать в гугле вопрос о глубине кратеров, то первые же ссылки будут именно о теории одинаковости и всей этой чепухе, так что непосвященному и далекому от астрономии человеку будет не так-то и просто разобраться где подвох.
Аноним 05/05/17 Птн 11:55:42 #24 №338143 
>>338139
>прочитал не отрываясь
Ебать ты превозмог. 23 двачестрочки осилил. Не отрываясь! Медальку тебе шоколадную за победу над прокрастинацией.
Аноним 05/05/17 Птн 12:34:31 #25 №338145 
>>338139
Продолжение будет попозже. Там довольно много материала и картинок, в один пост не запихнуть, думал за вчера все осилю, но смог родить только это. Посижу пару-тройку вечерков на даче, может высру еще.
Аноним 05/05/17 Птн 12:43:27 #26 №338146 
>>338143
>>338144
Просто тут 99% такой кал высерают, что максимум по диагонали через пост.
Аноним 05/05/17 Птн 12:54:58 #27 №338149 
>>338146
Тут некоторые время от времени предлагали собрать ответы на платиновые вопросы. Я так-то за эту идею, но никто нихуя делать не хочет, а одному мне западло. Собрать бы маленькую базу данных, что бы как у взрослых, со ссылками на нормальные источники, кОртинки и диаграммы плюс доступный текст. Можно было бы охунный научпоп, рожденный разумом улья тиснуть, для самых маленьких. Но как-то ходить из пейсикса в роскосый и обратно с ведрами говна анону, видимо, интереснее. Хорошо хоть Астро-тред внезапно ожил. Прям бальзам на яйца.
Аноним 05/05/17 Птн 13:24:18 #28 №338156 
1457564058875.png
1) Я постоянно слышу про воду на южном полюсе Луны и как её можно преобразовать в топливо. Где про это почитать можно? Ну в смысле как они добывать будут, извлекать из реголита, как её очистить, потом преобразовать в топливо и закачать без проблем в баки космического аппарата?

2) Может нужен общий Луна-тред для обсуждения всего связанного с нашим естественным спутником? В конце этого года туда китайцы садятся с возвратом, потом индусы и гугл, потом роскосый - не создавать же для каждой миссии свой тред.
Аноним 05/05/17 Птн 13:44:18 #29 №338158 
Какое самое реалистичное объяснение парадокса молодого Солнца?
Аноним 05/05/17 Птн 13:56:39 #30 №338162 
>>338158
> парадокса молодого Солнца
https://ru.wikipedia.org/wiki/%D0%9F%D0%B0%D1%80%D0%B0%D0%B4%D0%BE%D0%BA%D1%81_%D1%81%D0%BB%D0%B0%D0%B1%D0%BE%D0%B3%D0%BE_%D0%BC%D0%BE%D0%BB%D0%BE%D0%B4%D0%BE%D0%B3%D0%BE_%D0%A1%D0%BE%D0%BB%D0%BD%D1%86%D0%B0

http://www.membrana.ru/particle/14380
Аноним 05/05/17 Птн 14:00:16 #31 №338164 
>>338156
>Может нужен общий Луна-тред
С этими тредами одна беда. Нужно в него носить контент. Все время наполнять свежими новостями по теме, картиночками, темами для обсуждения. Иначе тред утоонет нахуй, и про него вспомнять через полтора года.
Вон один аутист с каменной жопой ведет тред про экзопланеты, с упорствомСизифа он не дает треду утонуть.
Тред создать - много ума не надо, дело 5 минут, вот наполнять его и сделать интересным для анона - другое дело. Так-то никто не запрещает же.
Аноним 05/05/17 Птн 15:01:51 #32 №338169 
F235RX1GXL6FHS4.MEDIUM.jpg
Что за хуйню я только что прочитал?

https://www.nasa.gov/directorates/spacetech/niac/2017_Phase_I_Phase_II/Mach_Effects_for_In_Space_Propulsion_Interstellar_Mission
Аноним 05/05/17 Птн 15:21:27 #33 №338174 
>>338173
Автор ищется и даже работа какая-то опубликована. У меня нет времени читать, посмотри что там

https://www.academia.edu/15592779/Theory_of_a_Mach_effects_thruster_II

https://www.researchgate.net/profile/Heidi_Fearn/publications
Аноним 05/05/17 Птн 15:22:58 #34 №338175 
>>338171
Вполне. Есть же космические станции, теперь представь, что эта же станция будет лежать на поверхности планеты, а не болтаться на орбите.
Дело в том, что они особо не нужны. Вот полностью автоматический завод-заправщик на орбите имеет куда больше смысла в качестве "космодрома подскока".
Аноним 05/05/17 Птн 15:25:03 #35 №338178 
>>338173
Алсо
https://en.wikipedia.org/wiki/Woodward_effect
Аноним 05/05/17 Птн 16:38:42 #36 №338185 
>>338180
>Как я распознать то смогу тру это или фрикачество?
Никак не можешь. Ты и смысл слова фрикачества не понимаешь.

Смысл в том, что бы проверить, работает ли принцип Маха и эффект Вудворда, или нет. Есть гипотеза и некоторое теоретическое обоснование. Теперь необходимо эмпирическим путем проверить эти обоснования и все. При положительном результате нужно понять, можно ли использовать вот это вот всё для прикладных целей в обозримом, пусть и не близком будущем. Обычное исследование.
Аноним 05/05/17 Птн 18:01:31 #37 №338195 
>>338037 (OP)
Как-то вышел с батей спор насчёт бытия ШВИТЫХ на луне. Так-то я в этом уверен, что сильнаярука там был, а он не верит. Приводил всякие доказательства, которые краем глаза где-то видел, он в свою очередь маневрировал всякой хуйнёй, что услышал по тель-а-визору. Так вот, мудрые аноны, может кто привести очень веские аргументы в пользу того, были ли пиндосы на луне иль наоборот, это заговор масонов
Аноним 05/05/17 Птн 18:06:04 #38 №338196 
>>338195
Хоть обчитайся: http://www.skeptik.net/conspir/moonhoax.htm
Аноним 05/05/17 Птн 18:21:46 #39 №338197 
>>338195
Вот тебе командировочный лист и таможенная декларация Базза Олдрина. Вишь документ: убыл, прибыл. Вывез-ввез.
Аноним 05/05/17 Птн 18:23:32 #40 №338198 
originalcad9f55c1d59b52993a979d0137a1382.jpg
oKtG14gwH10.jpg
picf1f5c935c3e553d2f955e3fab4c3ad66.jpg
>>338197
Отклеилось
Аноним 05/05/17 Птн 18:30:12 #41 №338200 
>>338198
>>338197
блгагодраю, анон
Аноним 05/05/17 Птн 18:40:13 #42 №338201 
>>338198
Будто это нельзя опровергнуть "аргументом" я тоже могу бумажку распечатать и подписей всяких понаставить

>>338195
Вот зачем тебе это? Оставь его в покое. Единственное чего ты добьёшься — посрёшься с батей.
Аноним 05/05/17 Птн 18:51:11 #43 №338202 
>>338201
Вот и я о том же: подделать можно все, что угодно, шизикам что не покажи у них все подделка и фальшивка, стоит ли связываться с поехавшими?
Аноним 05/05/17 Птн 22:48:55 #44 №338235 
orbitadesedna.gif
Короче эт самое, поиграл сейчас в один симулятор, и провёл некоторые тесты, если звезда массой в 1 Солнце пройдёт в районе где сейчас находится Седна, то вырвет из нашей солнечной системы все планеты до Нептуна включительно, если пройдёт звезда с массой в пять Солнц, то вырвет все планеты до Сатурна и распидорасит орбиту сатурна, если пройдёт звезда в десять масс Солнца, то вырвет все планеты до Сатурна и возможно сам Сатурн, и распидорасит все орбиты начиная с Сатурна и заканчивая Меркурием. Звезда в 50 масс Солнца вообще уничтожает нашу систему в принципе. Это правда?
Аноним 05/05/17 Птн 22:51:00 #45 №338236 
>>338235
>то вырвет из нашей солнечной системы все планеты до Нептуна
Я хотел сказать все объекты нашей Солнечной системы, грустный Плутон.жпг
Аноним 05/05/17 Птн 23:01:00 #46 №338238 
>>338235
что за симулятор? Но да, либо вырвет, либо на эксцентричные орбиты отправит и перемешает все в хаос.
Правда звезде сложно подкрасться незаметно. Их движения на миллионы лет можно предсказать.
Аноним 06/05/17 Суб 08:41:44 #47 №338259 
>>338195
1. Уголковый отражатель
2. Попроси у бати пруфы полета Гагарина Гагарина отслеживала NORAD, но вряд ли батя об этом знает
Аноним 06/05/17 Суб 09:04:59 #48 №338261 
>>338259
Что отражатель?
Аноним 06/05/17 Суб 09:31:15 #49 №338262 
>>338259
>1. Уголковый отражатель
Его батя не поймёт чё это за хуйня и как оно работает. Даже после объяснений.
>2. Попроси у бати пруфы полета Гагарина
ДА КАК ТЫ СМЕЕШЬ СОМНЕВАТЬСЯ В НАШЕЙ ИСТОРИИ!!!!11

Единственный способ переубедить его батю и прочих сторонников их там не было — свозить на луну и показать ёбаный модуль.
Аноним 06/05/17 Суб 11:15:34 #50 №338264 
>>338238
>Их движения на миллионы лет можно предсказать.
Нельзя, по крайней мере не при современной детальности наблюдений.

>>338235
Оценка близка к верной, полагаю. Но вообще-то Седна очень близко к Солнцу, по сравнению с размерами его полости Хилла. Практически внутреннее тело. А так-то говорят, что тысяч 70 лет назад сквозь внешнее облако Оорта (25 тыс а.е) прошла звезда Шольца, и ничего не сделала - далеко. https://arxiv.org/abs/1502.04655
Аноним 06/05/17 Суб 12:12:37 #51 №338269 
>>338262
>Единственный способ переубедить его батю и прочих сторонников их там не было
Вьебать в ебло пару раз.
/дискасс
Аноним 06/05/17 Суб 13:02:29 #52 №338274 
03082017-b4.jpg
>>338269
Достаточно одного, но для этого надо быть астронавтом.
Аноним 06/05/17 Суб 13:07:29 #53 №338277 
>>338264
> Нельзя, по крайней мере не при современной детальности наблюдений.
а на сколько?
Аноним 06/05/17 Суб 15:46:00 #54 №338302 
>>338264
> Нельзя, по крайней мере не при современной детальности наблюдений.
ну ближайшие звёзды посчитаны на сотни тысяч, gaia опять же хуярит и относительно скоро нахуярит ещё точнее и довольно большую сферу охватит
в чём проблема?
Аноним 06/05/17 Суб 16:53:28 #55 №338308 
14843212211470.png
https://www.youtube.com/watch?v=nDOxg5UY60E

В конце этого видео мужик поясняет (как он сам думает) что нужно строить ебическую катапульту с в вакууме и дрочит на Маска. Я знаю что у всяких шизанутых дядек есть свои предпочтения в запуливании грузов в космос, кто-то дрочит на лифт, кто-то строит гору, вот он хочет взять построить огромный метрополитен. Если тут где-то тред по теме, и достойна ли она обсуждения?
Аноним 06/05/17 Суб 18:59:29 #56 №338312 
>>338308
Если атмосферу с Земли сначала откачать, то почему бы и нет?
Аноним 06/05/17 Суб 19:06:19 #57 №338313 
>>338083
>>338084
Не знаю, может уже и не будет продолжения, но я жду, очень интересно.
Аноним 06/05/17 Суб 19:08:34 #58 №338314 
>>338312
Можно создавать вакуум на траектории снаряда.
Аноним 06/05/17 Суб 20:16:10 #59 №338316 
>>338308
в космосе ничего нет и непригоден, поэтому мировой капитализм считает, что капитальные затраты на удешевление запусков слишком велики.
и поэтому запусками и попытками удешевления запусков занимаются те кому больше всех надо - милитаристы и фрики-энтузиасты.
Аноним 06/05/17 Суб 22:05:54 #60 №338325 
Существуют ли открытые тройные лацертиды?
Аноним 06/05/17 Суб 22:38:52 #61 №338327 
>>338314
Как?
Аноним 06/05/17 Суб 22:48:55 #62 №338328 
>>338327
Ставим в снаряд насос, он откачивает спереди. Шах и мат.
Аноним 06/05/17 Суб 23:09:28 #63 №338329 
>>338328
Так тебе и метрополитен не нужен тогда, откачивай воздух и выкачивай его сзади.
Аноним 06/05/17 Суб 23:37:19 #64 №338333 
>>338329
Ты не поверишь
Аноним 06/05/17 Суб 23:47:33 #65 №338334 
>>338327
Жечь воздух лазором.
Аноним 07/05/17 Вск 00:55:59 #66 №338337 
>>338329
> откачивай воздух и выкачивай его сзади.
Шок! Сенсация! Малолетний гений с двача изобрел турбореактивный двигатель... читать_дальше
Аноним 07/05/17 Вск 04:16:29 #67 №338343 
Что будет если человека посадить на маленький астероид 500 метров в диаметре. Он будет видеть, что астероид закругляется? Если человек пойдёт по нему вперед, он потом будет стоять вверх ногами или ему будет всё время казаться, что он стоит прямо? Человек сможет спрыгнуть с астероида в космос или 500 метров это много для такого манёвра? А если меньше, например 100?
Аноним 07/05/17 Вск 05:33:07 #68 №338348 
>>338343
>Человек сможет спрыгнуть с астероида в космос или 500 метров это много для такого манёвра?
Вполне сможет. Скорее даже просто ходить не сможет без специальных приспособлений. Беглый расчет, даже если принять среднюю плотность такого объекта за 10 т/м3, дает первую космическую в размере 1,3 м/с, что в тысячу раз меньше, чем у Луны. Ну и если каменюка эта достаточно освещена, то ничто не помешает разглядеть очевидно большой показатель кривизны.
Аноним 07/05/17 Вск 05:51:09 #69 №338349 
>>338348
Даже ходить просто значит не сможет? Вот тебе и на...
Аноним 07/05/17 Вск 06:04:56 #70 №338350 
А что будет, если в мкс случится чп и человек оторвется от ремня и улетит в космос? Его полетят спасать? Не на чем же, значит не полетят, я думаю. Это правда, что в космосе не разлагаются трупы? В таком случае может ли этот человек, когда задохнётся, через миллиарды лет улететь в другую галактику? или с ним что-нибудь станет за такое время? Не считая распидорашивания в чьей-нибудь атмосфере, естественно.
Аноним 07/05/17 Вск 15:22:14 #71 №338361 
ISS-42EVA-1(d)TerryVirts.jpg
>>338350
>А что будет, если в мкс случится чп и человек оторвется от ремня и улетит в космос?
Уже было такое пару раз, только не улетал. У Сереброва например при ВКД на Мире в 1993, когда крепили ферму "Рапана", оторвался поручень, за который крепился страховочный фал. Когда осознал что некоторое время свободно плавал без страховки, чуть не охуел. Достаточно было двинуть рукой неудачно, и улетел бы.

Так-то на американские костюмы (EMU) с некоторых пор ставится внешняя реактивная система SAFER, пикрилейтед. Чисто теоретически, в случае отрыва можно подрулить, у неё где-то 3м/с запаса, если не ошибаюсь. У Орланов для этой цели есть УСК, его не юзают, когда-то пробовали более тяжелые установки, но они оказались неэргономичными.

Было множество экспериментов с "реактивными мотоциклами" на Мире и Шаттлах, позже их прекратили по разным причинам - отсутствие задач, неудобство, опасность случайного повреждения теплозащиты шаттла, опасность окончательного ухода от корабля/станции. Ещё у шаттлов предусматривался манёвр для подбора отцепившегося астронавта грузовым отсеком, капсулой так не словишь.

И да, если почитать результаты экспериментов, основная проблема - скорее визуально разглядеть, а не подобрать.
Аноним 07/05/17 Вск 15:23:12 #72 №338362 
>>338350
Он сойдет с орбиты через несколько месяцев.
Аноним 07/05/17 Вск 16:46:46 #73 №338363 
hubbleimagestscihp1714af6576x7614.jpg
https://www.nasa.gov/feature/goddard/2017/a-new-angle-on-two-spiral-galaxies-for-hubbles-27th-birthday/

Будут ли видны эти галактики другу другу невооруженным взглядом? Как если бы наша земля была в рукаве одной из них.

Межу ними 55 млн св. лет, та, что слева 87к лет в диаметре, та что справа 45к.
Антон 07/05/17 Вск 17:42:24 #74 №338364 
Перечитываю "Пять возрастов вселенной" Лафлина и Адамса, для большего осознания.
Подскажите что-то похожее космочан.

Хокинга читал, мимо плех.
Аноним 07/05/17 Вск 17:48:20 #75 №338365 
>>338364
Брайан Грин, Хокинг, Пенроуз.
Аноним 07/05/17 Вск 19:09:47 #76 №338375 
>>338363
> Межу ними 55 млн св. лет
андромеда более чем в десять раз ближе но видно её только в ебенях, хоть она и визуально больше луны
Аноним 07/05/17 Вск 19:43:14 #77 №338378 
луна.png
Есть нечто землеподобное 1 шт. и луноподобное 2 шт.
Барицентр на луноподобной орбите. Расстояние между лунами 1\2 от расстояния земли до барицентра.
т.е. перигелий каждой из лун 3\4 288350 от 384 467 км а афелий 5\4 483584.
Я так понимаю с поверхности сильно заметна разница видимого размера когда один почти в афелии другой почти в перигелии.
Наклон орбиты не более 5 градусов.
Вопрос: Насколько стабильна данная система? При максимальном грав. резонансе, какие будут орбитальные периоды?
Аноним 07/05/17 Вск 20:04:48 #78 №338380 
>>338378
> Насколько стабильна данная система?
нинасколько
Аноним 07/05/17 Вск 20:05:42 #79 №338381 
Много ли звёзд на звёздном небе в какой нибудь галактике с 10-100 трилионом звёзд?
Аноним 07/05/17 Вск 20:07:33 #80 №338382 
>>338380
На сколько нинасколько?
0,1 месяцев, 1 месяц, 12 месяцев, 120 миллиардов месяцев?
Аноним 07/05/17 Вск 20:08:19 #81 №338383 
>>338363
>34.2 МБ
Ты откуда такую принес, ирод?
Огромная, как галактики на ней.
Аноним 07/05/17 Вск 20:10:39 #82 №338384 
>>338363
Невооруженным взглядом и наша-то галактика нормально видна лишь в ебенях.
Аноним 07/05/17 Вск 20:12:05 #83 №338385 
>>338381
> Много ли звёзд на звёздном небе в какой нибудь галактике с 10-100 трилионом звёзд?
важен не размер, а плотность звёздного населения (шт на кубопарсек), туманностей и прочих молекулярных облаков, положение наблюдателя и спектр этого самого звёздного населения, тип галактики и ещё дохуя всего, например параметры атмосферы этого твоего звёздного неба
Аноним 07/05/17 Вск 20:13:35 #84 №338386 
>>338343
Астероид нинужин. На Луне из-за крайне неравномерной гравитации есть места, где гравитационная вертикаль отклоняется от геометрической на градусы.
Аноним 07/05/17 Вск 20:15:07 #85 №338387 
>>338382
установи юнивёрсал сэндбокс и попробуй в душе не ебу что это
Аноним 07/05/17 Вск 20:16:55 #86 №338389 
>>338386
Это называется гора или возвышенность в простонародье. Не?
Аноним 07/05/17 Вск 20:19:18 #87 №338390 
>>338389
это называется гравитационной аномалией( это такая хуйня из-за разной плотности пород)
хотя твой вариант тоже правильный, но там не градусы
Аноним 07/05/17 Вск 20:29:09 #88 №338391 
moon-grail-gravity-map.jpg
>>338389
Концентрации массы, масконы. Снаружи относительно плоско, но порода настолько неравномерной плотности, что в одном месте гравитация довольно прилично отличается от другого. Первые АМС, крутившиеся вокруг Луны, неожиданно быстро меняли орбиту или падали из-за этой хуйни, что было сюрпризом.
Аноним 07/05/17 Вск 20:30:39 #89 №338392 
>>338389
Имеется в виду разница между отвесом на нитке и направлением в центр планеты. Грузик на нитке будет висеть не точно вниз.
Аноним 08/05/17 Пнд 02:20:37 #90 №338409 
>>338387
>в душе не ебу что это
Охуенная поебень. Один-два вечера убить можно, как нехуй.
потом стер нахуй, читать полезнее
Аноним 08/05/17 Пнд 02:37:03 #91 №338410 
>>338409
Дваждую. У нас даже когда-то Univese Sandbox-тред был, но утонул не добравшись и до 100 постов вроде. Вылетала эта хуйня часто, ни у кого сил не хватало долго аутироввать. Сейчас поштабильнее стало?
Аноним 08/05/17 Пнд 02:59:36 #92 №338413 
>>338410
У меня не вылетало ни разу.
Аноним 08/05/17 Пнд 03:46:30 #93 №338419 
>>338410
Было джва треда.
С первым сэндбоксом, а потом со вторым. Анон быстро наигрался.
Аноним 08/05/17 Пнд 04:19:44 #94 №338422 
>>338418
>На сколько прилично? Если в среднем 0.16 G то к примеру в самом плотном будет 0.26 G а в самом рыхлом скажем 0.06 G?
Нет, ну не настолько большая разница, там градиент в доли процента. Надо смотреть результаты GRAIL, но вообще на память могу лишь вспомнить научпоп-фактоид, что на краю некоторых кратеров вертикаль наклонена.

>Из за чего такая неравномерность то?
Говорят, вот эта шняга виновата. https://ru.wikipedia.org/wiki/Поздняя_тяжёлая_бомбардировка В основном масконы совпадают с крупными кратерами, но не всегда и обычно со смещением.
Аноним 08/05/17 Пнд 05:43:10 #95 №338424 
Вот квантовая спутанность, да? Вот хули если один атом "1", то спутанный атом не обязательно "0"? Что, нельзя сделать так, что будет пара атомов с заранее известным условием, и если один атом будет нулём, то второй по любому будет единицей и наоборот? Все эти мансы со спинингом кажутся больше побочным логическим шумом в условии (т.е. мысленном эксперименте), который отвлекает от основной задачи.
Вот, сука, один атом движется влево, а другой вправо. Сменили направление первого атома, и второй должен автоматом поменять направление на противоположное, хули тут думать блять. Нихуя не понимаю.
Аноним 08/05/17 Пнд 14:52:14 #96 №338445 
Спейнасы, врываюсь с рекветом. Когда-то давно скачивал программу, вроде бы не спейсэнджин. Суть токова: открываешь её, она показывает текущее звездное небо из точки которую ты указываешь. Со всеми созвездиями, обьектами интересными и прочим. Из запоминающегося: можно было менять нижний фон, типа, с горы как будто сомтришь, с поверхности океана, с сатурна, с луны и пр. Куда-то проебал, а сейчас не могу вспомнить, как зовется эта программа. наверняка вы поняли о чем я. Если надо, могу ещё попытаться подробности вспомнить.
Аноним 08/05/17 Пнд 14:59:42 #97 №338447 
>>338445
Популярный-легендарный Stellarium.
Аноним 08/05/17 Пнд 15:09:41 #98 №338449 
>>338447
Спасибо большое, анон. Это оно, то что нужно.
Аноним 08/05/17 Пнд 20:54:02 #99 №338486 
>>338037 (OP)
Поясните за лучи-хуичи. Вот у видимого спектра имеются частицы — фотоны. А у радио что? А у рентгена?
Аноним 08/05/17 Пнд 20:56:05 #100 №338487 
hqdefault.jpg
>>338138
Где? Докажи!
Аноним 08/05/17 Пнд 21:13:05 #101 №338488 
>>338486
внезапно - фотоны
Аноним 08/05/17 Пнд 22:33:20 #102 №338498 
>>338119
При фотографировании космоса фокус всегда на бесконечность, независимо от расстояния
Аноним 08/05/17 Пнд 23:36:28 #103 №338508 
Кто смотрел вчера на луну? Возле неё была только одна звезда на небе. Что это?
Аноним 08/05/17 Пнд 23:37:00 #104 №338509 
>>338508
Юпитер
Аноним 09/05/17 Втр 00:04:08 #105 №338510 
>>338509
Больше на Венеру похоже.
Аноним 09/05/17 Втр 00:07:23 #106 №338511 
>>338508
сириус
Аноним 09/05/17 Втр 00:38:18 #107 №338516 
>>338508
Юпитер это. В бинокль до сих пор видно его спутники, хотя оппозиция была месяц назад.

>>338510
Какая Венера, она сейчас перед рассветом видна.
Аноним 09/05/17 Втр 03:59:38 #108 №338528 
1480892695.jpg
Есть уже всем набившая оскомину вундервафля. Почему все с ней так носятся и удивляются? Это же обычная "микроволновка" и то, что она работает ни кого удивлять не должно - штука излучает свч излучение, фотоны как известно обладают импульсом p=h·v/c, что учитывая закон сохранения импульса должно давать тягу в противоположную от излучения сторону.

Т.е штука должна работать и понятно как она работает и какой импульс дает. Собственно повторю вопрос - почему все удивляются, и перепроверяют?
Аноним 09/05/17 Втр 04:08:04 #109 №338529 
>>338528
По предъявам авторов, тяга больше чем давление излучения. Оттого собственно и носятся.
Аноним 09/05/17 Втр 05:09:03 #110 №338532 
>>338424
бамп вопросу
Аноним 09/05/17 Втр 06:04:18 #111 №338534 
>>338486
Что радиодиапазон, что рентген, что гамма, что видимый свет - все электромагнитные волны одной природы. И все это фотоны, по сути кванты эм-излучения. Просто энергия и длина волны сильно разнится.
https://ru.wikipedia.org/wiki/Электромагнитное_излучение
Аноним 09/05/17 Втр 06:47:28 #112 №338535 
Stellarium без интернетов работает?
Аноним 09/05/17 Втр 07:50:58 #113 №338536 
>>338535
Да.
Аноним 09/05/17 Втр 08:57:42 #114 №338540 
>>338536
У меня вылетал, значит проблема с мое стороны. Спасибо.
Аноним 09/05/17 Втр 09:43:00 #115 №338543 
Если вселенная бесконечна, то есть ли вероятность что у Земли есть свой клон, а у меня двойник?
Аноним 09/05/17 Втр 09:58:02 #116 №338544 
>>338543
Да, но для её реализации требуется время больше жизни Вселенной.
Аноним 09/05/17 Втр 10:44:37 #117 №338545 
>>338544
т.е больше жизни вселенной? Разъясни, почему больше жизни вселено, ведь вселенная бесконечна.
Аноним 09/05/17 Втр 10:54:15 #118 №338546 
>>338545
Вселенная большая, а вероятность маленькая.
Аноним 09/05/17 Втр 11:22:18 #119 №338549 
>>338535
может вылететь если вместо интернетов ты подключен к какой-то хуйне, было такое
Аноним 09/05/17 Втр 11:53:00 #120 №338550 
>>338544
Но если она бесконечна, то есть и вероятность что клон земли появился одновременно с нашей Землей.
Аноним 09/05/17 Втр 13:02:50 #121 №338556 
>>338545
Смотрите на дурака пока он в лесне убежал. Это мудило думает, что если умножит ноль на бесконечность, то получит единицу. Охуеть.
Аноним 09/05/17 Втр 14:13:47 #122 №338563 
>>338556
но у нас вероятность не ноль, потому дурак/мудак у нас ты.
Аноним 09/05/17 Втр 18:15:56 #123 №338603 
Отличаются ли чем-то ракеты для вывода экипажа и мёртвого груза равной массы? Почему пилотируемый Союз выводит одна ракета, а точно такой же беспилотный Прогресс - совсем другая?
Аноним 09/05/17 Втр 18:17:05 #124 №338604 
>>338603
Ну, кроме наличия САС. Может разный допустимый уровень перегрузок или что ещё.
Аноним 09/05/17 Втр 18:21:22 #125 №338605 
>>338603
Обтекатели ещё разные из-за этой башни САС, вроде всё.
Аноним 09/05/17 Втр 19:19:38 #126 №338610 
Поясните как работают парниковые газы. Они же вроде задерживают и переизлучают часть ИК излучения от нагретой планетки обратно к ней? Но тогда ведь получается, что чем больше таких газов в атмосфере, тем меньше ИК излучения будет достигать поверхности планеты, и тем холоднее она будет?
Аноним 09/05/17 Втр 19:29:41 #127 №338613 
Чёт я не понял, если в космосе ваккуум и тела могут обмениваться тепловой энергией только при непосредственном контакте, то как космические корабли охлаждают кабину, нагретую солнечным излучением?
Аноним 09/05/17 Втр 19:31:23 #128 №338615 
>>338613
Можно ещё излучением охлаждаться. На корабли и станции ставят радиаторы.
Аноним 09/05/17 Втр 19:33:44 #129 №338616 
>>338615
Это я знаю, но куда радиаторы отводят тепло? То есть даже если он в тени, конвекции нету
Аноним 09/05/17 Втр 19:35:43 #130 №338618 
>>338616
Конвекция нинужна. В радиаторы затекает теплоноситель, они нагреваются и излучают в ИК диапазоне, тем самым охлаждаясь.
Аноним 09/05/17 Втр 19:39:17 #131 №338619 
Understood.jpg
>>338618
Благодарю
Аноним 09/05/17 Втр 20:23:26 #132 №338627 
image.png
>>338610
Не выпускают ИК в космос, тепло остается под парниковой шапкой. Планета не охлаждается, а нагревается.
Аноним 09/05/17 Втр 20:34:57 #133 №338630 
>>338627
>Не выпускают ИК в космос
Но тогда и не должны пускать ИК из космоса, и планета должна остывать, не?
Аноним 09/05/17 Втр 20:53:07 #134 №338634 
>>338630
Не, ты не догнал. Из космоса к ним приходит энергия в разных диапазонах, а от Земли, когда она отдает тепло, только в ИК.
Аноним 09/05/17 Втр 21:09:16 #135 №338635 
>>338634
Ну греет-то в основном как раз ИК, который эти газы и задерживают. Да, обычный видимый свет проходит, но ИК-то блочится на подлете, получается.
Аноним 09/05/17 Втр 21:21:05 #136 №338636 
>>338635
Ты решил включить идиота?
Аноним 09/05/17 Втр 21:56:45 #137 №338638 
>>338636
Нет, просто хочу докопаться до истины.
Аноним 10/05/17 Срд 08:21:04 #138 №338657 
Что происходит?
Аноним 10/05/17 Срд 11:26:48 #139 №338668 
>>338198
Охуеть. Декларация космонавта. Таможня для космических путешественников.
"лицехватов везете? а он со справкой от ветеринара? прививки все есть?"
"реголита не больше килограмма на человека"
"ну все, турист, ты приехал. видишь, собака лает? ща тепловой щит вскроем. лучше сразу говори, что спайс везешь, может до старости из тюрячки выйдешь"
Приземляется капсула в Венесуэле, а ей добро на посадку не дают. Пиздуй, мол, к себе, капиталист проклятый.
Аноним 10/05/17 Срд 11:31:44 #140 №338669 
>>338668
Ну а как иначе? Из страны выезжал же. И обратно въезжал. Похуй что в бочке-это уже частности. Хорошо хоть ТС по кобылам налогом не обложили.
Аноним 10/05/17 Срд 11:35:07 #141 №338671 
>>338669
>ТС по кобылам налогом не обложили
Потрачено.
Аноним 10/05/17 Срд 11:41:30 #142 №338674 
>>338673
> кстати почему на Меркурий ровер не отвезли?
не будет работать, алсо дорого возить на меркурий
Аноним 10/05/17 Срд 11:46:47 #143 №338676 
>>338673
>почему на Меркурий ровер не отвезли
Думаю, из-за жары. Электроника вся охуеет. Хотя там же как с Луной у нас, одной стороной к Солнцу всегда. Можно кататься на холодке.
Значит просто нет задач. Самый печальный и всеми забытый кусок камня в системе. Даже Плутон популярнее.
>>338674
Радиация? Поглядел, на темной стороне комфортные -100.
Ох, лол. На темной стороне. РИТЭГ надо везти через адово пекло на подлетах.
Аноним 10/05/17 Срд 11:49:07 #144 №338677 
>>338675
днём жарко, лепестроника не может вроде, и фольгой толком не обмотаешься, разве что на полюса ебошить
ночью холодно и долго и холодно и долго... без ритега накроется всё
Аноним 10/05/17 Срд 11:50:11 #145 №338678 
>>338676
> Хотя там же как с Луной у нас, одной стороной к Солнцу всегда
ёбу дал?
Аноним 10/05/17 Срд 11:52:07 #146 №338680 
>>338679
> не хватает урана\плутония на Земле
не нерабатывают ибо дорого и экология
шоб было дешевле - нужно больше хуярить, а задач нет
Аноним 10/05/17 Срд 11:54:44 #147 №338681 
>>338563
Манька даже не может в математику. Охуенно, чо.
Аноним 10/05/17 Срд 11:57:50 #148 №338682 
>>338678
>Период обращения: 88 дней
>Продолжительность суток: 58 д 15 ч 30 мин
Таки да. Набрехал.
>>338680
Разве не из-за политики их перестали делать?
Со своим электричеством не было б тех проблем на комете.
Аноним 10/05/17 Срд 12:01:45 #149 №338684 
>>338682
> Разве не из-за политики их перестали делать?
политика, экология, нераспространение ядерных материалов вроде ещё, мало задач вне космоса
Аноним 10/05/17 Срд 12:37:32 #150 №338690 
>>338679
> не хватает урана\плутония на Земле для массового использования этих штук
Нет задач для этого говна на земле тащемта. Урана и прочих изотопов - до ебени матери.
Аноним 10/05/17 Срд 21:29:09 #151 №338746 
>>338673
>кстати почему на Меркурий ровер не отвезли
Не приоритетно. Да и там только что орбитер отлетал - неразумно посылать ровер первым же аппаратом, без разведки.

>>338675
По результатам MESSENGER'а на полюс предлагали гипотетическую миссию в постоянно затенённые кратеры.
Аноним 10/05/17 Срд 22:12:35 #152 №338754 
NewHorizonsRTGinPHSF.jpg
>>338684
Плюс вес и габариты. Где-то читал что ЕКА отказались от идеи использовать ритеги для Филея ещё на стадии проэктирования т.к. аппарат получался слишком дохуя большим и тяжёлым.
Аноним 10/05/17 Срд 23:08:58 #153 №338764 
Каковы уравнения движения по эллиптической орбите в полярных координатах?
Аноним 10/05/17 Срд 23:17:26 #154 №338765 
>>338754
Есть мелкие. Там чисто европейские заёбы
Аноним 10/05/17 Срд 23:57:23 #155 №338766 
>>338658
И в чём смысл этого всего? Если и так известно, что в конце всему наступит тепловая смерть, зачем страдать ради ничего в итоге? Почему мы не можем пойти против природы, если всё равно стремимся к пути техногенного развития и подавления инстинктов? Зачем нужны полёты в другие системы, ради чего эти превозмогания?
Аноним 11/05/17 Чтв 00:24:01 #156 №338770 
>>338766
прост))
Аноним 11/05/17 Чтв 08:25:14 #157 №338789 
>>338787
> или же абсолютно все 40 вариантов из области чистейшего "Си-фи" и маня гипотез?
это
Аноним 11/05/17 Чтв 13:22:50 #158 №338807 
Тут часто спрашивали как система сформировалась
https://www.youtube.com/watch?v=sdwp4Jklj88&feature=youtu.be
Аноним 11/05/17 Чтв 16:06:05 #159 №338820 
>>338681
ты даун или притворяешься? в школе теорему вероятности не проходил? для тебя 0.01 * вселенная = 0 ? ты долбаёба кусок? P = m/n ни о чём не говорит?
Аноним 11/05/17 Чтв 20:07:11 #160 №338855 
>>338820
Так вот кто у нас прогуливал матан.
Аноним 11/05/17 Чтв 21:26:02 #161 №338874 
>>338779
И зачем? Смерть совершеннее этих конвульсий. Если человечество платит огромным количеством жертв и страданий за миги удовольствия от удовлетворения любопытства, то никакого смысла в самих сверхпознаниях нет. Бессмертный бестелесный разум будет ждать вечность безумия.
Аноним 11/05/17 Чтв 23:46:48 #162 №338885 
Как получилось так, что луна находится именно на таком расстоянии, что размеры её диска идентичны солнечному?
Аноним 12/05/17 Птн 01:25:23 #163 №338890 
>>338885
Тебе повезло родиться в такое время.
Аноним 12/05/17 Птн 04:06:53 #164 №338897 
>>338543
бесконечное количество клонов в таком случае.
Аноним 12/05/17 Птн 05:09:44 #165 №338900 
>>338553
на астрофоруме такой бред прочитал?
Аноним 12/05/17 Птн 08:29:19 #166 №338905 
что делает такое количество ебучего сайфача в ттв?
Аноним 12/05/17 Птн 08:41:51 #167 №338906 
>>338905
Очевидно, что в спейсаче сидят более научно подкованные господа, нежели кукаретики из сайфача.
Аноним 12/05/17 Птн 12:31:49 #168 №338915 
>>338820
> 0.01
Ты откуда эту цифру высрал, чмо?
Аноним 12/05/17 Птн 16:48:55 #169 №338941 
>>338855
>пук
пиздуй в школу, уебан.
>>338915
из твоей матери высрал, дауна кусок. не можешь в образность? у нас есть вероятность, но никто не знает какая она, тебе будет легче, если я напишу 0.000000000000000000000000000000000000000000000000000000000000000000000000000000000001.
Похуй сколько цифр после нуля, умножить на вселенную, а вселенная это - 100% или 1, то будет 110^-n, то всё равно результат будет больше нуля.
Единственная проблема, которую можно наблюдать это то, что в этих 100% нет колна/двойника, но, вселенная постоянно расширяется, то есть 100%
n, и из-за постоянного разрастания есть шанс на наличие клона, а если такой имеется, то вероятность уже не будет 0.
Уебаны, пиздуйте в школу и готовьтесь к эгэ
Аноним 12/05/17 Птн 16:53:42 #170 №338943 
>>338941
забыл про
1
10^-n.
клона.

быстрофикс
Аноним 12/05/17 Птн 18:56:33 #171 №338948 
>>338941
>вселенная это - 100% или 1, то будет 110^-n, то всё равно результат будет больше нуля.
Ну иди мне найди двойника на плутоне или около сверхновой.

>но, вселенная постоянно расширяется, то есть 100%n, и из-за постоянного разрастания есть шанс на наличие клона
А количество материи у тебя тоже постоянно возрастает, дебил?

уносите его
Аноним 12/05/17 Птн 19:08:49 #172 №338949 
>>338941
>>пук
Кто тут уебан еще? Нахуй ты так пишешь? Питурд дохуя? Так пиздуй в /b/, тут таким не рады.


мимокрокодил
Аноним 12/05/17 Птн 21:23:38 #173 №338959 
>>338948
Что эта сверхманевренная хуйня делает на спейсаче?? Вместо конструктивной критики:
>ну иди найди на плутоне
А то, что вопрос был "возможно ли во вселенной" тебя не волнует? Я говорил про то, что клон будет на плутоне или может ты перестанешь долбится в глаза и посмотришь, что я говорил про вселенную в целом?
>материи
>не возрастает
А у тебя типо звезды/планеты из планетарных туманностей и прочих космических лабуд не образуются? Вселенная расширяется в вакуум? Повторяю, пиздуй нахуй в школу на уроки физики и математики.

>>338949
Ну так и мимокрокодиль дальше нахуй. По существу сказать нечего потому решил доебаться до манеры речи?
Аноним 12/05/17 Птн 21:35:50 #174 №338961 
Подскажите формулу для определения скорости космического аппарата на орбите, например, Земли.
Разве все сводится к 2пиR/T?
Аноним 12/05/17 Птн 21:51:11 #175 №338962 
>>338959
Какую тебе еще конструктивную критику, ты пишешь какую-то хуйню, вселенная это 100%, течение времени увеличивает вероятности. Иди почитай про максимально возможное квантовых состояний, если хочешь прикинуть наличие твоих двойников.
Аноним 13/05/17 Суб 03:54:05 #176 №338968 
>>338902
мы что, живем в объемном горизонте событий чд? не, ну под это еще можно формулы подогнать, это я более-менее понял, но как и почему он должен с ускорением расширяться в 4-ное пространство вокруг, я не понял. в каком контексте рассматривать статью, в научном, философском или юмористическом?
Аноним 13/05/17 Суб 04:07:13 #177 №338970 
>>338969
вот и я не разбираюсь, и понять где там бред, а где более-менее обоснованная с научной точки зрения реальность, я как ни старался, не смог. по моему, в первую очередь, это какие-то дебри в голове автора, а не во вселенной и механизмах которые он описывает (потому что их просто нет).
Аноним 13/05/17 Суб 07:47:28 #178 №338973 
>>338959
Какой агрессивный шкоьник, вы только гляньте.
Я тебя обмудка ещё раз спрошу. С какого хуя ты вообще решил что вероятность, хоть бесконечно малая но есть? Тебе так КАЖЕТСЯ? Откровение было? Ты просто мудак необучаемый, которому вселеная что-то там должна потому что яскозал!
Аноним 13/05/17 Суб 10:14:45 #179 №338978 
14546148582100.jpg
Чому по луне не шастают роверы типа спиритов и опортьюнитов и не делают хойрез фотачки для быдла? Это же гораздо проще и дешевле закинуть эту поебень на луну, а не ебучий марс.
Аноним 13/05/17 Суб 10:32:52 #180 №338981 
>>338980
Можно же пустить роверы по обраточке
Аноним 13/05/17 Суб 10:34:12 #181 №338982 
>>338980
шо то кусок камня, шо это. так тут хоть земляшку пофоткать можно. подозреваю, что не пускают из-за радиации и перепада темрератур
Аноним 13/05/17 Суб 10:53:01 #182 №338984 
>>338977
Можно, если ты умеешь в сайфайные способы конвертации энергии в полезную для тебя форму.
Аноним 13/05/17 Суб 10:58:01 #183 №338986 
>>338962
что-то я почитал, и ничего не нашел, лучше тыкни носом в статью или где читать.

>>338973
школотролль порвался, пиздуй в школу, тебе ещё 11 класс проходить, учить теорию вероятности, а потом и в вузе.
Аноним 13/05/17 Суб 11:07:58 #184 №338988 
>>338987
Так тебе только елистричество нужно вырабатывать чтоли? Так бы и писал. Внезапно энергии бывает дохуища видов.
Аноним 13/05/17 Суб 11:08:41 #185 №338989 
>>338987
>чем отличается
Длиной волны.
Аноним 13/05/17 Суб 13:06:06 #186 №338994 
>>338977
>>338979
>>338985
> 1-4
да, но необязательно это будет рентабельно
> 5
да, уже, миллиарды лет, упадёт, улетит, столкнётся, нерентабельно, закон сохранения энергии
> 6
пока не придумали, вообще могут и сами столкнуться
можно ещё и белые в кучу
> 7
материя и так туда неплохо поступает
> 8
сомнительно, как и от бабаха
> 9-12
сайфай какой-то
распад протона - гипотеза
> 13
Среди позднего М-класса тоже есть коричневые карлики, там уже местами вопросы веры начинаются
> как тусклые красные, только термоядерная реакция не постоянная, а переменная
тут ты очень правильно понял, но у коричневых карланов основной сорт топлива другой и с его выгоранием и остыванием шансы продолжить горение падают к нулю
Аноним 13/05/17 Суб 13:37:31 #187 №338996 
>>338986
>тебе ещё 11 класс проходить
Ясно. Понятно.
Аноним 13/05/17 Суб 14:16:52 #188 №338997 
>>338986
>тыкни носом
Начни с этого. Ты же знаешь английский, образованный ты наш?
https://www.youtube.com/watch?v=8GEebx72-qs
Аноним 13/05/17 Суб 15:46:22 #189 №339004 
>>339002
>Что плохого в средней температуре реликтового излучения в 280 по Кельвину, а не убогие 2.8 ? Не одной ледяной планеты во всей вселенной!
Только, если лично для тебя. Прожаренный ты хуйню нести меньше/совсем не будешь.
Аноним 13/05/17 Суб 16:06:17 #190 №339006 
Можно на Луне совершить rocketjump стреляя автоматом калашникова под ноги?
Аноним 13/05/17 Суб 16:27:05 #191 №339007 
>>339006
Да.
Аноним 13/05/17 Суб 16:28:46 #192 №339009 
>>339006
Энергии очереди одного автомата калашникова на Луне хватит только на ~25кг массы. Поэтому если ты 50-килограммовый дрищ и будет в каждой руке держать по автомату, то может тогда и сможешь.
Аноним 13/05/17 Суб 16:47:08 #193 №339014 
Астронавты Аполлонов имели ограниченное количество видео- и фото- пленки?

Можно ли установить на Луне камеру, которая будет стримить видео прямо на Землю и постоянно будет держать её в фокусе?
Аноним 13/05/17 Суб 16:48:45 #194 №339016 
>>338985
Коричневые карлики отличаются не по температуре, а по массе. Если масса больше, чем 75 масс Юпитера, то это красный карлик, а спектральный класс может быть даже L2, и при этом известны коричневые карлики с классом M8
Аноним 13/05/17 Суб 16:50:23 #195 №339017 
Можно ли расколоть нейтронную звезду и каким-то образом вывести "нейтрониум" из родного гравитационного колодца?
Аноним 13/05/17 Суб 17:04:10 #196 №339018 
>>338997
Как можно в 2017 не знать ангельский? И да, знаю, спасибо за ссылку
Аноним 13/05/17 Суб 17:11:27 #197 №339019 
>>338999
> тритий\дейтерий?
можно и просто на дейтерии
для протия слишком большой реактор нужен, размером со звезду
так что проще закармливать им белые карланы
Аноним 13/05/17 Суб 17:14:20 #198 №339020 
14271936840330.jpg
Когда и кем впервые было сделано предположение/утверждение, что человек не сможет дышать в космосе? Как я понимаю, это же не какое то "базовое" знание, на вроде того, что водой дышать нельзя, огонь обжигает, а снег холодный.
Аноним 13/05/17 Суб 17:17:56 #199 №339022 
>>339016
> масса больше, чем 75 масс Юпитера
одной массы недостаточно, нужно ещё тяжелых элементов для катализа и гравитационный нагрев шоб загорелось
рандомная хуйня на 75 масс юпа не обязательно красный карлан
Аноним 13/05/17 Суб 17:19:23 #200 №339023 
>>339017
нет, алсо нейтрониум бабахнет при отрыве от родного колодца
Аноним 13/05/17 Суб 17:21:03 #201 №339026 
>>339014
> Астронавты Аполлонов имели ограниченное количество видео- и фото- пленки?
да
> Можно ли установить на Луне камеру, которая будет стримить видео прямо на Землю и постоянно будет держать её в фокусе?
можно
Аноним 13/05/17 Суб 17:22:51 #202 №339027 
>>339014
> Астронавты Аполлонов имели ограниченное количество видео- и фото- пленки?
А как ты представляешь себе неограниченное количество?
> Можно ли установить на Луне камеру, которая будет стримить видео прямо на Землю и постоянно будет держать её в фокусе?
Да, можно, кроме финансовых никаких особенных препятствий тому нет.
Аноним 13/05/17 Суб 17:32:17 #203 №339029 
>>339028
> аккуратно
> на очередные миллиарды лет?
нет
может быть как-нибудь импульсно, на тысячи-миллионы, а может и нет
напиши какому нибудь Сурдину, лул
чет тяжелый сайфай пошел
Аноним 13/05/17 Суб 17:44:36 #204 №339030 
>>339014
>Можно ли установить на Луне камеру

Можно, до первого метеорита.
Аноним 13/05/17 Суб 17:47:44 #205 №339031 
>>339030
до очередного перепада температур
Аноним 13/05/17 Суб 17:49:27 #206 №339032 
>>339026
>>339027
Я неправильно вопрос сформулировал. Может у Аполлонов был какой-то лимит, который они достигли или были близки, а потому не могли снимать все подряд, боясь что им не хватит пленки. А следующая экспедиция на Луну сможет снимать хоть круглые сутки и не иметь никаких ограничений. Сняли несколько гигабайт 8K HD футадж? Отправили на Землю, и снимай дальше.
Я по крайней мере надеюсь на это. До сих пор даже на МКС не всегда есть 4К видео, а на выходы в открытый космос видео тоже говно. Я блять надеюсь что в 2025-х на Луну отправят нормальные камеры, а не какое-то говно, которое только будет давать пищу для теоретиков заговора.
Аноним 13/05/17 Суб 20:06:14 #207 №339038 
>>339028
Нет, не может. Он насасывает водород с компаньона до образования слоя водорода на своей поверхности определённой толщины, после чего случается бадабум и запускается CNO цикл, в результате часть водорода перерабатывается, а часть выбрасывается нахуй термоядерным взрывом. Потом все повторяется. Накопление-взрыв-сброс. Такие звезды называются новыми.
Аноним 13/05/17 Суб 20:07:59 #208 №339039 
>>339032
А нахуя? Какая практическая ценность у этого?
Аноним 13/05/17 Суб 20:45:27 #209 №339041 
Если вселенная расширяется, то получается, что все расширяется вместе с ней, каждый атом, протон, время ускоряется?
Аноним 13/05/17 Суб 21:04:29 #210 №339044 
>>339041
подтверждён только разлёт галактик пропорционально расстоянию
всё остальное - пока что гипотеза
Аноним 13/05/17 Суб 21:39:51 #211 №339052 
>>339039
Блять, тебе в роскосый тред с такими вопросами. Рашка бы точно если бы послала космонавтов на луну, то не дала бы им фотоаппаратов и камер. По той же причине:
>А нахуя? Какая практическая ценность у этого?
Аноним 13/05/17 Суб 23:15:31 #212 №339059 
>>339052
Иди нахуй пиздорвань. Видево ему подавай в 8к. А нахуя объяснить не может. Медиадитё залетное.
Аноним 13/05/17 Суб 23:51:11 #213 №339060 
>>338890
Но даже раньше, когда она была немного ближе размер диска соответствовал солнечному. Это не может быть совпадением.
Аноним 14/05/17 Вск 00:03:15 #214 №339061 
>>339060
> Это не может быть совпадением.
ты не можешь быть человеком
Аноним 14/05/17 Вск 02:14:17 #215 №339064 
Как вообще всё возможно? Есть какое-то объяснение?
Аноним 14/05/17 Вск 05:35:28 #216 №339066 
>>339041
>время ускоряется
С чего бы вдруг? Да и атомы-протоны не расширяются. На них действует достаточно стабилизирующих сил. Расширяется только пространство, "пуустота" в бытовом понимании.
Аноним 14/05/17 Вск 07:45:43 #217 №339068 
Вот маловероятный сценарий.
Газовый гигант состоящий преимущественно из кислорода и покрыт тонким слоем инертного газа. В глубине же содержится сжиженный углеводород. Спустя время система врезается в водородное облако. Водород обволакивает планету. Спустя время инертный газ рассеивается. О и Аш вступает во взрывообразную реакцию. Тем временем в центре планеты образуется давление и водород содержащийся в нефте, и высвободившийся из неё путём нагрева.(а нефть не загорелась потому что тоже покрыта инертным веществом) вступает в термоядерный синтез.

Какие именно элементы этой афёры возможны? А что надо сделать что-бы исправить и осуществить таки подрыв планеты естественным путём? Что будет при таком подрыве? Разлетится или даже незаметно будет?
Аноним 14/05/17 Вск 08:35:05 #218 №339070 
> в нефте
Аноним 14/05/17 Вск 10:37:40 #219 №339073 
httpsvkcomebankotop (1).gif
Как себя чувствуют и веду космосычи с околоземной орбитальной станции во время вспышек на солнце, когда солнечный ветер достигает, собственно, станций?
Чё там по самочувствию людей?
Чё там по состоянию электроники?
Аноним 14/05/17 Вск 11:35:03 #220 №339074 
>>339068
> Газовый гигант состоящий преимущественно из кислорода и покрыт тонким слоем инертного газа. В глубине же содержится сжиженный углеводород.
углеводород рванёт ещё при формировании планеты
> Спустя время инертный газ рассеивается
нет

алсо вся эта хуйня очень плотная и невозможная впринципе

>>339072
контекст не как что-то плохое, но...
Аноним 14/05/17 Вск 12:07:53 #221 №339077 
>>339076
Да, возможен. примерно такие сценарии рассматриваются для аэродинамического торможения межпланетных аппаратов, только там они после этого выходят на очень эллиптическую орбиту и дальше постепенно дотормаживают похожим манером.
Аноним 14/05/17 Вск 12:19:28 #222 №339078 
>>339071
> распылить
что это значит в твоём понимании?
> Метеориты
лул
> Астероиды
пододвинет далеко, мелкие может вынести из системы
> плонетки
не образуются в системе сверхновой
Аноним 14/05/17 Вск 12:21:00 #223 №339079 
>>339064
> Как вообще всё возможно? Есть какое-то объяснение?
прост
Аноним 14/05/17 Вск 12:34:41 #224 №339088 
blob
Если на Земле пересохнут океаны, она приобретет со временем полностью круглую форму?
Аноним 14/05/17 Вск 12:44:06 #225 №339090 
ПДФ - копия.jpg
>>339076
Вполне возможен. Но тогда желательно иметь ебическую скорость к моменту ныряния в атмосферу, чтобы оная не смогла существенно повлиять на камушек так, чтобы тот в итогу гуфнулся прямо на планету эту. С поверхности такой Земли (вряд ли с Венеры будет видно) это будет выглядеть сначала как обычая звезда, потом, по мере приближения, как яркая звезда уровня Юпитера, потом уровня Сириуса, потом как маленькая но очень яркая горящая точка. А потом будет нырок в атмосферу. Во-первых, по законам физики, перед главный стеройдом и за ним, немного погодя, будут лететь микрокамушки, которые в 99,(9)% сгорят в атмосфере, но могу упасть на поверхность или вызвать сгущение облаков - станут центром формирования газопылевых скоплений и водяной коденсации, но это лирическое отступление. Первое соприкосновение с атмосферой может вызвать раскат грома, по крайней мере в верхних слоях, ибо аккустичекая волна от резкого удара, все дела. Затем начнёт камень гореть, может быть аки луна, а может и ярче, оставляя за собой расходящийся вот такой < или сходящийся вот такой > след.
При этом само тело будет иметь перед собой ударную волну, визуально отделяющую само тело от внешней среды. Проходя нижайшую к поверхности точку своей траектории, каменоид будет горче ярче пукана японцев под Хиросимой. Возможно, буквально будет как второе солнце, только при этом с грохотом и рёвом. А следом за пролётом, вместе с настижением аккустической ударной волны поверхности, ещё ебически шандархнет громом и, к тому е, поднимется ураганный ветер. А потом данный гость улетит и хуй с ним.
Как-то так.
Аноним 14/05/17 Вск 13:06:35 #226 №339092 
>>339088
Это фейковая картинка.
Аноним 14/05/17 Вск 13:16:28 #227 №339093 
>>339092
И давно нас ЕКА фейками кормит?
Аноним 14/05/17 Вск 13:19:51 #228 №339094 
>>339088
это гипертрофированная карта высот
реальная круглее
Аноним 14/05/17 Вск 14:17:50 #229 №339097 
>>339071
Распылить не сможет ничего, только расплавить. Также все крупные тела станут несколько тяжелее, т.к. притянут к себе немного звездного вещества. Орбиты изменятся, т.к. центральное тело теряет значительную массу, всё что крутится вокруг неё становится слишком быстрым для своей орбиты. Орбита вытягивается на разные величины, вплоть до гиперболической (индивидуально для каждой звезды и планеты)
Аноним 14/05/17 Вск 14:19:57 #230 №339098 
>>339092
>>339094
Тем не менее, "вжимают" ли океаны землю "внутрь"? Выгнется земля обратно если океаны исчезнут?

Мимодругойанон
Аноним 14/05/17 Вск 14:23:05 #231 №339099 
>>339098
Океаны легче чем земля под ними, ничего особенно не изменится. Алсо земля не резина, и океаны "исчезают" только в /sf/
Аноним 14/05/17 Вск 14:50:40 #232 №339106 
>>339099
На марсе исчезли например
Откуда тогда образовались впадины океанов если их не вода продавила?
Аноним 14/05/17 Вск 15:42:46 #233 №339113 
>>339111
> Он же ещё вполне работоспособен
топлива уже почти нихуя нет
> учёных больше не интересуют данные, что он может собрать? Или остро хочется получить данные из атмосферы Сатурна?
не настолько утрированно, но ты расписал верно
> И типа таки занёс земных микробов туда..
вопрос веры
Аноним 14/05/17 Вск 16:02:17 #234 №339115 
>>339106
Ты как-то перепутал причину и следствия. Это не океан продавил, а заполнил низины.
Аноним 14/05/17 Вск 16:03:12 #235 №339116 
БАМП >>339073
Аноним 14/05/17 Вск 16:13:27 #236 №339120 
>>339073
Нормально себя чувствуют. Сидят и чай из самовара пьют с баранками. Хуле им будет? Они же внутри магнитосферы, а не снаружи.
Аноним 14/05/17 Вск 16:21:02 #237 №339121 
>>339120
> Сидят и чай из самовара пьют с баранками
> на орбитальной станции
ты охуенен
Аноним 14/05/17 Вск 17:47:38 #238 №339125 
>>339120
То есть, орбитальная станция - это дом тех самых мужиков с бревном в те редкие минуты покоя, когда врагошар не нападает?
Аноним 14/05/17 Вск 17:51:23 #239 №339126 
>>339120
А если отправить орбитальную станцию навернуть пару низкоорбитальных кругов вокруг Луны и, соответственно, лететь до Луны и обратно?
Что защитит мужиков, из самовар и баранки на лунной орбите?
А если отправят на этой станции к Венере или к Марсу? Какие средства защиты от ебических вспышек человечество имеет?
Аноним 14/05/17 Вск 18:04:38 #240 №339129 
>>339126
Никаких не имеет. Аполлоны летали на 2 недели перекрестясь и уповая на низкую активность Солнца. На счёт более длительных путешествий пока не понятно. Есть разные варианты. Создать специальное уёбище убежище небольшого размера, где можно пересидеть за бронешторкой переждать, пока все не утихнет. Короче, мы работаем над этим.
Аноним 14/05/17 Вск 18:22:11 #241 №339131 
ujcu5x9axU.jpg
>>339125
Мужики с бревном? Врагошар?
Аноним 14/05/17 Вск 18:40:43 #242 №339132 
>>339131
загугли "мужики с бревном", "врагошар". чё как маленький?
Аноним 14/05/17 Вск 21:55:37 #243 №339139 
08bfafdec0d07ebf0d71f8vklw.png
>>339131
http://arhivach.org/thread/57083/#83622485


Мужики офигенны. Особенно их страсть к изобретательству, ведь они готовы на все чтобы остановить с помощью бревна враго-шар. А еще они уважают традиции - в основе конструкции всегда лежит старое, проверенное бревно, не отходят от канона ведения войны.

Посмотрите только на их лица. Какие они собранные, отважные. А еще они настоящие друзья: крепкий и жилистый мужик с маленькой головой держит тяжелый конец, а долговязый направляет атаку бревна на технически подкованного противника. Т.е. на лицо строгое распределение обязанностей и взаимовыручка.

Я представляю, как после битвы, они аккуратно кладут бревно на заснеженное поле, садятся на него и неспешно пьют чай из полкового самовара, беззвучно шевеля своими черными ртами и переодически взмахивая безпальцевыми культяпками. Такие вот простые и суровые. У них наверное и имена такие же - суровые и простые, Серега и Васек. Серега пониже, постарше, одна нога у него короче другой, зато массивная и позволяет ему выделывать немыслимые па в бою и на отдыхе. Зато у него менее выразительная мимика человека, повидавшего все. Васек же более подвижен, не утратил пыл молодости, все ему интересно, но в лице его также читается сталь, только ей еще надо немножко закалиться.

Знаете, я бы и сам хотел быть таким вот мужиком, чтобы у меня в мире было только два интереса: победа и бревно. Ну и верный друг, куда уж без него
Аноним 15/05/17 Пнд 04:36:42 #244 №339152 
>>339139
Был в том треде. Он ещё кажется начинался как меч vs катана.
Аноним 15/05/17 Пнд 04:45:40 #245 №339153 
1492713326097.jpg
>>338037 (OP)
>Тред тупых вопросов
Ок.
У меня вопросы по поводу распидорашивания земли Земли космическими каменюками. Правильно ли я понимаю, что раз земля это шар из раскалённой магмы покрытый тонкой коркой из застывшего вещества, то астероид движущийся с не слишком большой скоростью, но вполне внушительной массой проломит поверхность планеты и утонет в ней? И что будет с луной в такой ситуации?
Аноним 15/05/17 Пнд 08:20:37 #246 №339158 
>>339155
Есть карликовая планета Хаумеа, оче похожа на первый пик. Для звезд - вряд ли, из-за достаточно больших размеров они слишком медленно вращаются обычно. Для белых карликов или нейтронных звезд, которые быстро вращаются из-за своих малых размеров, это больше похоже на правду, но беглый поиск показывает, что там будет порядка 10% разницы между экваториальным и полярным диаметрами.
Аноним 15/05/17 Пнд 08:24:24 #247 №339159 
>>339155
Ну и нагрев звезд в любом случае идет из центра а не с поверхности, потому такой структуры, как ты нарисовал, не получится никак.
Аноним 15/05/17 Пнд 08:31:30 #248 №339160 
>>338198
Мне кажется, в /law надо вбросить тему "Правовой статус исследовательских аппаратов вне Земли", хотя там небось всё просто: принадлежат государству в лице соответствующего органа, заказавшего у изготовителя
Аноним 15/05/17 Пнд 09:47:57 #249 №339162 
14455463340420.jpg
>>339154
Ок. Теперь действительно много глупых вопросов, если не ответишь - всё нормально.

Поехали.
Тогда, при каких условиях земля будет распидорашена на кусочки, или правильнее капли? Когда это случится куски Земли разлетятся в разные стороны, притянутся друг другу соединившись в целое тело, или может расползутся по орбите образуя пояс?
Как Луна поведёт себя при разрушении Земли? Будучи самым массивным телом она может стать ядром новой планеты или лишившись "опоры" отправиться в свободное плаванье? Если может, то как это будет происходить, может ли получиться так, что луна притянув несколько крупных капель окажется достаточно массивной, что бы собрать остальные в пояс вокруг себя, сольются ли они в одно тело или остыв образуют кольца? Если нет то что с ней будет?

PS И насколько вообще могут быть точны ответы на такие вопросы при отсутствии вводных данных о параметрах астероида?

PPS Пока печатал понял, что проще будет мне самому прочить какую нибудь литературу о планетообразовании, чем доёбывать людей, не посоветуешь чего?
Аноним 15/05/17 Пнд 09:54:48 #250 №339163 
>>339160
Там сорт оф морское право. А по факту рулит тот, кто может обеспечить статус кво.
Таким образом если на Луне валяются ботинки астронавта, то они де юре принадлежат США. По факту, если ты сможешь туда прилететь и забрать их тебе некому будет помешать.
Аноним 15/05/17 Пнд 10:19:37 #251 №339164 
0U3R0oYhUkM.jpg
14660694646760.jpg
14947913688692.jpg
IlPZrYNhANg.jpg
>>339154
> 1. Гость является кометой и насыщен льдом разного типа, а не камнями\металлами
Если основной состав врагошара - пылепар, он просто взорвётся в атмосфере, вызвав большой бум и громкий, который может Землю обойти не раз, вспышку яркую, атмосферу покорёжит (если прям большой, но не слишком) и в радиусе взрывной волны где-то на сутки - вряд ли дольше, будут распидорашены все стабильные горизонтальные ветряные потоки, останутся только примитивные вертикальные, то есть возможны смерчи, ебические шторма/тайфун (опять же частиц камня = эпицентры конденсации), потом облачность, а при более менее устканившей потоковости ветров, будет по планете огромный анти/циклон осадки виде снега-дождя разбрасывать, размазывая пыль по поверхности. Вероятно радиационного и биохомического заражения от таких осадков рассматривать не будем. Но можешь посмотреть фильм Эволюция (2001)
> 2. Скорость полёта астероида относительно Земли СЛИШКОМ высокое, сопротивление атмосферы оказалось катастрофически сильным.
Больше скорость - больше сопротивления и трения воздухом, больше физического нагрева -> ярче горит, большее число молекул поверхности вступает в реакцию с атмосферным бульоном. Б0льшие шансы взорваться, ибо для камня плотность растёт. Если ты прыгнешь в воду с высоты двух-трёх метров, тебе особо ничего не будет. А вот если с высот пары сот километров, для тебя вода будет как земля бадум-тссс.
> 3. Астероид нырнул в атмосферу и вынурнул, но его успело затормозить так, что он вышел на вытянутую эллиптическую, в итоге спустя н дней снова входит в атмосферу с приличной скоростью не до конца остыв с предыдущего захода и... ПЕРЕГРЕВ БАБАХ
Тогда много осколков появится, часть сгорит, часть упадёт на поверхность, а часть дальеш полетит, но небольшая.
Аноним 15/05/17 Пнд 10:22:48 #252 №339165 
2f7faa799f[1].jpg
>>339155
Есть космический пельмень
https://naked-science.ru/article/sci/ryadom-s-saturnom-nashli-gigantskiy
Аноним 15/05/17 Пнд 10:43:50 #253 №339167 
>>339165
вот бы его с космомаянезиком космозахавать
Аноним 15/05/17 Пнд 12:12:53 #254 №339172 
>>339162
> Тогда, при каких условиях земля будет распидорашена на кусочки
масса примерно с марс, скорость сотни-тысячи километров в секунду примерно почувствовал, без камукторных симуляций - никак
если меньше - всё сольётся
> Если может, то как это будет происходить, может ли получиться так, что луна притянув несколько крупных капель
или частично распределиться в луну

Аноним 15/05/17 Пнд 13:47:03 #255 №339185 
14948309357950.png
Много ли на близкой и не очень околоземной орбите маленьких астероидов уровня 3~ метров в диаметре?
Есть ли где-нибудь карта всех мелких тел в условной близости к Земле онлайн? Пускай и троянцы, но только на такие далёкие, чтоб Л4 или Л5, а поближе
Аноним 15/05/17 Пнд 14:18:35 #256 №339188 
>>339185
Всяких полно. А мелочь даже толком не считают.
https://cneos.jpl.nasa.gov/ca/
Аноним 15/05/17 Пнд 16:45:31 #257 №339201 
>>339176
>>339175
>>339172
Спасибо, анон.
ЛИТЕРАТУРА Аноним 15/05/17 Пнд 20:57:33 #258 №339224 
Друзья, подскажите самую годную литературу по астрономии.
Аноним 15/05/17 Пнд 21:04:44 #259 №339227 
>>339224
>>306847 (OP)
Аноним 16/05/17 Втр 09:07:50 #260 №339321 
>>339313
Ну ты уже сам нашел все, сомнительно, чтобы нашлись звезды, превышающие радиус той же VY CMa аж на порядок.

Добавлю только, что масса звезды тоже сомнительна. Насколько я помню, сейчас теоретическим пределом массы звезды считается что-то в районе 120-150 M☉, в Большом Магеллановом облаке обнаружены ( http://arxiv.org/abs/1007.3284 ) несколько исключительно массивных звёзд, из которых только три превышают лимит в 150 M☉, и лишь одна сейчас имеет массу 265 M☉, т.е. это штука исключительно редкая. Считается, ЕМНИП, что они были образованы слиянием нескольких звёзд поменьше, что возможно только в плотных скоплениях.
Аноним 16/05/17 Втр 09:29:28 #261 №339328 
>>339323
> летел со скоростью 500-2000 километров в секунду, но затормозил и влетел в Солнечную.
лул, полтора нолика сифи
> На каком расстоянии от Земли его смогут засечь
если он не будет излучать на землюху и иметь оchемалое альбедо - то где-то на ноо, может чуть выше
Аноним 16/05/17 Втр 10:11:01 #262 №339334 
>>339329
> термоядерные\ядерные движки
на исследовательском амс - сифи
шумы несовместимые с наукой
алсо набирать такую скорость шоб потом тормозить - тоже попахивает
алсо и топливо не резиновое
Аноним 16/05/17 Втр 10:59:38 #263 №339340 
>>339337
> залететь в звёздную систему, собрать данные потом манёвр под гравитацией звезды и к следующей и так далее?
не одними гравиманёврами, но суть ты уловил, топливо и его возможности ограничены и даже ядерники и ионники - не панацея
Аноним 16/05/17 Втр 15:49:06 #264 №339388 
>>339387
тёмной хуйнёй именуют то что можно померять но невозможно или практически невозможно исследовать
Аноним 16/05/17 Втр 17:24:33 #265 №339413 
14947777968240.jpg
если вот прямо сейчас по щучьему веленью, по моему хотенью на луне окажется километровый слой чернозёма (не с земли, а просто), допустим 2-3 километровый слой воды там, где низины подразумевают моря и океаны, запихнуть всякие грибные, мховые, папоротниковые и прочие леса флорой и всякие жуки и примитивные млекопитающие/рептилии в качестве фауны, а ещё атмосферу километров на 10-100, сколько столетий или тысячелетий пройдёт, прежде чем такой живой луне наступят пиздарики?
пиздарики уровня 1: 99%-100% фауны dies
пиздарики уровня 2: флора dies
пиздарики уровня 3: атмосфера выветривает
пиздарики уровня 4: больше нет воды (по любым причинам)
пиздарики уровня 5 ultimate tier: жизнепригодная почва курвится до непригодного уровня а-ля луна сейчас
Аноним 16/05/17 Втр 18:06:41 #266 №339420 
>>339413
Вода и газы сразу испарятся (ну, почти сразу) без них вся живность помрет. От температуры в -270 ..+дохуя тоже помрет практически сразу. А еще радиация есть и возможна какая нибудь ядовитая хуита в грунте как перхлораты на Марсе.
Аноним 16/05/17 Втр 18:09:00 #267 №339422 
fastest-rotating-star.jpg
vfts-102.jpg
>>339155
https://en.wikipedia.org/wiki/VFTS_102
Аноним 16/05/17 Втр 18:11:10 #268 №339423 
>>339420
не сразу же. от минимум 500 лет до максимум 5000 лет будет умирать такая живая луна
кто считал же что-то подобное пару десятков тредов назад
Аноним 16/05/17 Втр 18:14:10 #269 №339424 
>>339420
атмосфера, знаешь что это?
вся описанная тобой хуйня произойдёт только при сильном её истощении, что произойдёт не раньше сотни-тысячи(минимум) лет(считали уже, в старых тредах есть ссылки)
алсо радиации там понт, через слой грунта не профонит
Аноним 16/05/17 Втр 18:15:26 #270 №339425 
>>339155
https://en.wikipedia.org/wiki/SS_433
https://en.wikipedia.org/wiki/PSR_J1748-2446ad
Аноним 16/05/17 Втр 18:23:02 #271 №339426 
gJex-mexqoY.jpg
>>339423
>>339424
Дайте ссылку на рассчёты, хочу заценить
Аноним 16/05/17 Втр 18:24:09 #272 №339427 
>>338037 (OP)
Последний телескоп наса слишком широк, как его выводить будут, на атласе?
Аноним 16/05/17 Втр 18:28:00 #273 №339428 
>>339427
JWST? На Ариане, и он будет после запуска раскладываться.

https://www.youtube.com/watch?v=bTxLAGchWnA
Аноним 16/05/17 Втр 19:39:42 #274 №339449 
Есть книга или набор статей про астрономические эксперименты в домашних условиях? Чтобы как в древности сидя на камнях голой жопой размер земли посчитать и расстояние до солнца измерить.
Аноним 16/05/17 Втр 19:48:40 #275 №339452 
>>339449
орбитальная механика
опыты по определению гравитационной постоянной
наблюдательная астрономия в древности
строительство мегалитов
Аноним 16/05/17 Втр 19:50:13 #276 №339454 
>>339452
Это не книги и не статьи. Я не хочу из разных источников по крошкам собирать инфу.
Аноним 16/05/17 Втр 20:08:01 #277 №339460 
>>339454
Тогда иди нахуй, нехотелка, если 200 страниц осилить не можешь.
Аноним 16/05/17 Втр 20:08:35 #278 №339461 
>>339428
Спасибо.
Аноним 16/05/17 Втр 20:32:52 #279 №339462 
Гора в лен области(60 широта) наклонённая к плоскости земли на 60 градусов, так что перпендикулярна лучам солнца.
Означает ли это, что за секунду на неё падает столько же фотонов сколько и на экваторе\тропиках?
Аноним 16/05/17 Втр 20:36:37 #280 №339465 
>>339319
Если учесть, что при столкновении у нас бы был новый катархей это прекрасная перспектива, максимум на пару сотен тысяч лет.
Аноним 16/05/17 Втр 20:41:06 #281 №339466 
>>339462
Атмосферу забыл.
Аноним 16/05/17 Втр 20:56:21 #282 №339469 
1448380920163222742.jpg
>>339319
> Если шарик окунётся в атмосферу будет жарко.
Такого размера шар порвётся приливными, зачем ты пиздишь. Небольшая приливная сила, помноженная на огромную массу породы краёв шара, дадут усилие на разрыв, которое не выдержит ни один материал.
Аноним 16/05/17 Втр 21:10:40 #283 №339477 
>>339155
Ты только что блицар
Аноним 17/05/17 Срд 00:01:58 #284 №339510 
gaVhfUaTQW51cZ-PvR3cFQ.jpg
ЕБАНИСТИЧЕСКАЯ ХУЙНЯ ДВАЧ! Я УЖЕ ПРОСТО ЗАЕБАЛСЯ РЫТЬ ЭТОТ ВОПРОС, призываю всех любителей астрономии, геодезии и космоса.

При наблюдении зенитных растояний с астрономического универсала, и как результат, при высчитывании широты пункта наблюдения, нахуя нам собственно уровень и поправка за наклон?? каким хуем уровень влияет на широту, если он там не в центре и т.д. я очень запутался, нид хелп
Аноним 17/05/17 Срд 03:56:49 #285 №339519 
>>339512
Юневерс сндбокс 2 всего лишь игра с примерными параметрами, в котором вполне возможна ситуация, где какой-нибудь плутон пройдёт сквозь какой-либо Ио не потеряв в массе, пробыв почти под его поверхностью.

Что бы почти не упасть на планету такой ахуевший шар должен иметь релятивистскую скорость. А коснувшись атмосферы он её разогреет до охулиарда градусов и по сути опять расплавит кору.
Аноним 17/05/17 Срд 06:03:19 #286 №339522 
>>339510
Определение астрономической широты помнишь?
астрономическая широта — угол между отвесной линией (перпендикуляром к геоиду) и плоскостью экватора Земли. Равна угловой высоте полюса мира. Определяется с помощью астрономических наблюдений
Вот уровень как раз определяет направление к центру масс планеты. Он тебе дает отвесную линию. Раньше по старинке как раз отвес на веревочке использовали, но там долго ждать, пока он успокоится. Естественно отклонение уровня/отвеса напрямую влияет на точность определения широты, для этого и поправка, чтобы влияние ошибок снизить и точность повысить. Это ведь очевидно, может ты что другое подразумевал?
Аноним 17/05/17 Срд 07:38:24 #287 №339524 
Чисто теоретически, насколько большой может быть звезда и есть ли объективные ограничения по поводу её размеров?
Аноним 17/05/17 Срд 08:10:08 #288 №339532 
>>339524
>>339321
Аноним 17/05/17 Срд 09:42:51 #289 №339534 
>>339387
Официальная наука обясняет такое явление как ТЭ и ТМ так : ХЗ

Моё же мнение, что это поток времени раздвигающий булки вселенной и в этой концепции нет такого понятия как контрреакция, а лишь иллюзия.
Аноним 17/05/17 Срд 09:47:49 #290 №339535 
>>339534
> Моё же мнение
иди нахуй
Аноним 17/05/17 Срд 11:04:04 #291 №339540 
>>339539
конечно, просто нужно яснее давать понять что твоё мнение основанно на "примерно почувствовал" и прочих фантазиях
хотя бы в ттв
Аноним 17/05/17 Срд 11:13:46 #292 №339542 
>>339541
название этого треда
Аноним 17/05/17 Срд 13:24:57 #293 №339556 
>>339512
За пол оборота она вытянется в кучу камней. Надо пони что с такой массой шар не сопротивляется на разрыв сколько—нибудь значимо, а значит приобретет форму, как отдельно летящие его части по разным орбитам (ибо удалены от центра на разное расстояние) с его скоростью на момент пересечения предела роша
Аноним 17/05/17 Срд 14:30:12 #294 №339559 
delfin.gif
Берём Юпитер, Сатурн, Уран, Нептун, всех их спутники, Эриду, Маке-Маке и сталкиваем. Получаем маленькую звезду уровня 2mass j0523-1403. Скорее всего красную.
Если не хватает массы магическим путём сталкиваем с этой кучей ещё что-нибудь, кучу астероидов, Планету Х, Аллаха, сидячего деда с собакой и даже твою мамку - выполняем условие "ЕСЛИ".
Теперь у нас СС состоит из 2х светил - Солнца и... как будут называть вторую звезду? Дуо? Люкс? Мини?
Насколько сильно это поколеблет внешние границы СС? Допустим, Меркурий, Венера, Земля с Луной и Марс с Деймосом и Фобосом + пояс астероидов остаются "под юрисдикцией" Солнца, а система Плутон-Харон - будет вращаться вокруг второй звезды.
Но как такая звезда будет смотреться с Земли? Столкновения планеты с Земли будут только видеться, или ещё как-нибудь ощущаться? Ну там, гравиволны будут шатать трубу твоего дома, дожди будут идти сильные
Где будет находиться центр масс системы Солнца А и Солнца Б?
Будет ли ночью намного ярче, раз теперь не только Луна, но и бывшая кучка планет светит с неба?
Аноним 17/05/17 Срд 14:35:26 #295 №339560 
>>339559
> 2mass j0523-1403
Упс, я опечатался. имелась в виду система TRAPPIST-1 (также 2MASS J23062928-0502285 или EPIC 246199087)
Аноним 17/05/17 Срд 14:43:22 #296 №339564 
>>339561
>ловить солнечные лучи
>переизлучать их к земляшке
У меня КПД от такого.
Аноним 17/05/17 Срд 14:44:08 #297 №339565 
>>339562
Да говорю же: не важно как. Меня интересует, что в итоге.
Не о сифятине думай, а об условиях поставленной задачи
Аноним 17/05/17 Срд 15:00:37 #298 №339568 
>>339561
>что за дичь на втором пике
https://www.google.ru/url?sa=t&source=web&rct=j&url=https://www.nasa.gov/pdf/716070main_Mankins_2011_PhI_SPS_Alpha.pdf
Аноним 17/05/17 Срд 15:14:54 #299 №339573 
tumblrnkcgksgZjZ1qewacoo11280.jpg
>>339570
Спасибо, Аноний.
Ждём вашего репортажа с места событий.
Аноним 17/05/17 Срд 15:35:24 #300 №339574 
Поясните за объект Торна-Житкова
Аноним 17/05/17 Срд 15:57:16 #301 №339576 
>>339574
Не Житкова, а Житков (это женщина).

Что тебе именно пояснить? Вот статья на Википедии, если после её прочтения будет непонятно - возвращайся: https://ru.wikipedia.org/wiki/%D0%9E%D0%B1%D1%8A%D0%B5%D0%BA%D1%82_%D0%A2%D0%BE%D1%80%D0%BD%D0%B0_%E2%80%94_%D0%96%D0%B8%D1%82%D0%BA%D0%BE%D0%B2
Аноним 17/05/17 Срд 15:58:40 #302 №339577 
14945158314890.gif
>>339574
>Поясните за объект Торна-Житкова
Аноним 17/05/17 Срд 16:17:18 #303 №339580 
Как плотность черной дыры может быть меньше плотности воздуха? https://www.youtube.com/watch?v=Y_MFbNZVbGs
Аноним 17/05/17 Срд 16:29:15 #304 №339582 
oByM85Y5Xc.jpg
Два космонавта в открытом космосе свободно дрейфуют.
У каждого из них есть оружие. У одного - пистолет. У другого - лук и стрелы.
Тот, что выстрелит из пистолета, отлетит назад (без поправки на вращение) относительно направления вылета пули. Причём, пуля будет лететь с 0,5 своей скорости, а 0,5 скорости передастся космонавту.
А будет ли аналогичное с тем, который выстрелит из лука? По-моему, нет, потому что тетева поглотит энергию, и космонавт будет неподвижен (или почти) относительно своего до-выстрелового положения, а стрела с почти полной скоростью полетит вперде.
Получается, в космосе, менее затратно и более целесообразно использовать стрелы тетевной тяги, чем жидко- и твердотопливные движки.
РАЗВЕ НЕТ?
Аноним 17/05/17 Срд 16:32:56 #305 №339583 
>>339582
А хули затвор пистолета тогда не поглотит?
Аноним 17/05/17 Срд 16:33:18 #306 №339584 
За счёт чего нейтронные звёзды обладают такой охуенной скоростью вращения? Жидкое ядро и большой начальный импульс в начале жизни? (после взрыва сверхновой)
Аноним 17/05/17 Срд 16:34:00 #307 №339585 
>>339582
Так он ведь не выстрелит из пистолета, т.к капсюль не подорвет порох. Разве нет?
Аноним 17/05/17 Срд 16:39:14 #308 №339589 
1490809063130342103.jpg
lnpaIdMXpx0.jpg
tumblro1r64evbRg1uea7pwo1500.gif
tumblronc08vEHfU1qzhjh2o11280.jpg
как получить планету с водой, песком, какой-нибудь пускай и примитивнейшей но жизнью, но чтобы полностью вообще без атмосферы или атмосферы ультрамаленькой и гиперразряженной, измеряемой в сотни метром максимум и процентами плотности плутонианской атмосферы
Аноним 17/05/17 Срд 16:40:09 #309 №339590 
>>339589
> с водой
> без атмосферы
никак
Аноним 17/05/17 Срд 16:41:16 #310 №339591 
>>339589
> роцентами плотности плутонианской атмосферы
тоже никак вода не останется
Аноним 17/05/17 Срд 16:43:23 #311 №339594 
>>339593
> Как вариант подлёдная жизнь ещё.. Сверху льда атмосферы нет..
лёд станет атмосферой
Аноним 17/05/17 Срд 16:45:40 #312 №339595 
>>339592
вполне, осталось только герметичность
Аноним 17/05/17 Срд 16:48:37 #313 №339597 
>>339596
> планету
Аноним 17/05/17 Срд 16:54:38 #314 №339599 
>>339593
алсо что будет с требуемым песочком?
Аноним 17/05/17 Срд 16:58:15 #315 №339600 
>>339591
ага, как же. не останется. возьми планету размером с венеру или землю, и просто помести в район от марса до юпитера на орбиту своей звезды идентичной солнцу. тогда вода под силой тяжести не улетит
Аноним 17/05/17 Срд 16:59:07 #316 №339601 
>>339585
Пороху не нужен кислород, он уже в его составе.
>>339582
Пуля полетит с той же скоростью что и на земле с поправкой на отсутствие сопротивления воздуха.
Аноним 17/05/17 Срд 17:00:02 #317 №339602 
>>339600
вода испарится
Аноним 17/05/17 Срд 17:00:34 #318 №339603 
>>339599
а что с ним не так? песок - мелкодисперсный остаток какого-нибудь камня, получившийся в результате точения подводными/подлёдными течениями, если уж жизнь какой-нибудь рачковой-кабачковой не завезли
Аноним 17/05/17 Срд 17:03:52 #319 №339604 
>>339598
>>339587
Nien! Nein! Nien!
Не поместить Траппист-1 где-то на задворках СС, а столкнуть Юпитер с Сатурном, с Ураном, с Нептуном, с Хаумеей, с Маке-маке, с Эридой тгде-то в пределах орбит Нептуна и Урана, чтобы из из столкновения получилось что-то, похожее на Траписта.
Алсо, у тебя на видео получается, что Трапист - более яркая и жаркая звезда, потому что у него зона комфорта намного дальше, чем у более большого и жаркого Солнца. Это серьёзный прогон в прописании условий. Как у тебя малыш Траппист стал таким горячим?
Аноним 17/05/17 Срд 17:05:34 #320 №339606 
>>339602
куда она тебе испарится при температурах ниже нуля по цельсию? в анус бога морей нептуна огромным ледяным батплагом?
Аноним 17/05/17 Срд 17:14:34 #321 №339610 
>>339606
В пизду твоей мамаши. Гугли температуру кипения воды при нулевом давлении.
Аноним 17/05/17 Срд 17:30:07 #322 №339612 
file (5).gif
>>339610
> Гугли температуру кипения воды при нулевом давлении.
> При атмосферном давлении, стремящемся к нулю по цельсию, температура кипения воды составляе 6,7 градусов по цельсию
> при температурах ниже нуля по цельсию
> ниже нуля по цельсию
Аноним 17/05/17 Срд 17:35:06 #323 №339613 
3P-lNNUWNW8.jpg
>>339608
>>339609
>>339607
Ну допустим не между Ураном и Нептуном, а между Сатурном и Юпитером. Всё равно же в первые годы столкновения всё должно гореть ярче пукана Сычова, когда ему мамка рассказывает про успехи Ерохина.
По крайней мере, есть у меня такое предположение.
Тем более для более реалистичного ЕСЛИ можно всё облако Оорта с астероидным поясом скормить этой бурлящей пост-планетарной куче, и туда ещё Планету Икс впендюрить. Её же там просчитали как юпитероподобную, да?
Аноним 17/05/17 Срд 18:32:33 #324 №339615 
>>339612
Хули ты рожу скривил, петух?
Аноним 17/05/17 Срд 18:39:26 #325 №339616 
>>339600
Вода при нулевом давлении в жидком виде вообще не существует. Если это просто глыба льда, то со временем она съебет путем сублимации. Чем ниже температура льда и чем меньше на нее попадает света, тем дольше будет происходить этот процесс.
Аноним 17/05/17 Срд 19:12:59 #326 №339617 
>>339615
ты - планета из заданных условий, у тебя в самые жаркие дни, когда ты ближе всего к своей звезде, температура воды не поднимает выше -50 или даже -20 по цельсию. много у тебя жидкой воды и воды возгонкой съебёт с поверхности сраного тебя, спиздив сраные джоули?
и куда эта вода будет мигрировать?
с такой водой будет то же, что со светом от ядра чёрной дыры: излучаться-то фотоны излучаются, да вот только далеко убежать не могут и падают обратно на поверхность.
>>339616
сколько лет солнечной системе?
много там с европы и энцелада воды съебало?
Аноним 17/05/17 Срд 19:39:33 #327 №339620 
>>339617
>Жидкая вода
>-50
>Атмосферы нет
Съеби уже, петух мелкобуквенный.

>много там с европы и энцелада воды съебало?
Много жизни нашел на Европе или Энцеладе? Алсо, там температура поверхности -180 градусов, ебанулся совсем?

Аноним 17/05/17 Срд 20:20:58 #328 №339623 
>>339620
на поверхности лёд. а через пару километров уже вода, потому что тройная точка смещается под давлением. но главное - внешняя часть воды замкнута и никуда не возгоняется из-за льда
Аноним 18/05/17 Чтв 08:29:34 #329 №339641 
>>339584
>большой начальный импульс в начале жизни
Именно. Плюс когда сбрасывается внешняя оболочка, то внутреннее давление, препятствовавшее гравитации пропадает и все начинает хорошенько сжиматься, то за счет уменьшения радиуса вращения звезда тоже неплохо так разгоняется.
Аноним 18/05/17 Чтв 08:41:53 #330 №339642 
>>339639
А, то есть худо-бедно как Уран.
Ну что ж, Траппистой такая кучамала может и станет, но или будет тусклой, или таки яркой, но коротковременно - не больше периода в пару человеческих жизней.
Аноним 18/05/17 Чтв 08:56:47 #331 №339644 
>>339387
притягивается все в локальном плане, а разлетается в глобальном.
Аноним 18/05/17 Чтв 10:04:18 #332 №339651 
>>339647
По-моему, это манятеория и манявидение художника без каких-либо обоснований, А то, что там написано BEST, так это может за графон кто-нибудь написал. Или сам рисовальщик решил себя похвалисть и себе в картинку такое текст вставил.

[ЭТОТ КОММЕНТАРИЙ ПРИЗНАН ЛУЧШИМ В ТРЕДЕ]
Аноним 18/05/17 Чтв 10:57:02 #333 №339653 
>>339647
Как конкретно взрываются сверхновые, до мелочей, идут жаркие споры и единого понимания до сих пор нет. В общих чертах механизм ясен, но дьявол, как всегда в деталях. Ударные волны-это ударные волны разлетающегося говна, при чем тут гравитационные волны вообще?
Картинка красивая, наслаждайся, а то в реальных симуляциях на суперкомпьютерах получается типа "кто-то наблевал на раковую опухоль". Оно понятно: люди делом заняты, им не до эстетики, они над механизмом процесса ломают голову.
Аноним 18/05/17 Чтв 13:32:43 #334 №339661 
>>339656
>визуализицию грави волн
Ну ёбана!

>а те искажения как рябь такая, ну видно же ну
В первый раз на телефоне смотрел, не заметил.

>и если вы говорите что сие творение говнецо с точки зрения реализма
С точки зрения реализма очень трудно судить, остатки сверхновых на ранней стадии формирования еще никто не видел, а то, что получилось в финале показывают в этой симуляции уже изнутри.

>что тру сверхновые взрываются онли сферой расширяющегося света
Сверхновые взрываются далеко не симметрично. Это опять таки заметно по остаткам сверхновых. Тоже самое показывают почти все симуляции.

>Насколько это всё реалистично?
Фоток туманностей в инторнете дохуя, посмотри и сравни че там в спейсэнджине. В целом инженер подходит очень ответственно к моделям в своем планетарии.
Аноним 18/05/17 Чтв 15:05:37 #335 №339664 
14948023144790.jpg
>>339658
> ffffFFFFUUUUU
Интересное названое для зведопланеты.

А вообще K P A C N B 0.
Сидел залипал кучу времени.
Но пока я смотрел, понял вот что: сели ИРЛ из-за чего-либо указанные планеты столкнутся, даже если только Сатурн с Юпитером, Сатурн намотает на вал юпитерианского динамо, его кольца и обломки планетарной массы поедут делать супербольшое кольцо на НОО которое визуально увеличит Юпитетурн и сзделает его видимым с Земли даже невооружённым глазом, вокруг уже Юпитетурна, ну их спутники полезу в юпитетурнианский центр масс. Одно только это - как я сейчас думаю - будет полыхать и само-по себе ярко, и визуально от объёма будет казаться больше, и ещё солнышком посланный свет это всё отразит и увеличит. А если на вечную вписку в Юпитетурн прикатятся два брата-акробата легендарны Уран и его не менее популярный приятель Нептун, тогда Юпитетуранептун получит нечто, аналогично аккреционному диску звёзд, а ещё вокруг самой планеты будет ещё объёмное, тем более визуально, краш-облако, изнутри подсвечиваемое газом жаром столкновения, радиацией и светом полыхания, а снаружи - Солнцем. В общем, выглядеть будет такая херовина очень большой. Я думаю, от пятой части до трети визуального обьъёма полнолуния для наблюдателя на Земле.

Что думаете насчёт данной теории, господа?
Аноним 18/05/17 Чтв 15:28:28 #336 №339667 
1486552740421.jpg
Вот я постоянно слышу, что мы состоим из звездной пыли. А можно точно определить из какой сверхновой эта звездная пыль? Можно ли как-то научно узнать, что вот эта вот звезда, которая сейчас стала белым карликом, взорвалась и создала облако, которое продрейфовав образовало Солнце и планету Землю? Там одна сверхновая создала достаточно материала, или несколько? Если их несколько, то можно точно определить их количество и конкретные звезды?
Аноним 18/05/17 Чтв 15:30:28 #337 №339668 
Если джет от нейтронной звезды направить на чисто углеродную планету, то обстреливая её нейтронами, электронами и прочим говном нейтронная звезда изменит её состав? Будет происходить что-то вроде синтеза короткоживущих элементов, как в ускорителях?
Аноним 18/05/17 Чтв 16:05:43 #338 №339669 
>>339667
Нет, нельзя. Во-первых невозможно определить точное место зарождения СС. Невозможно определить нахождение и миграцию сверхновых (да, очевидно не одной и не в один день ебанувших). А так же у сверхновых нет уникальных подписей. Все изотопы во Вселенной одинаковые. Ну и в конце концов, после взрыва сверхновых образуются не белые карлики, а нейтронные звезды и чёрные дыры.
Так что нет, нельзя.
Аноним 18/05/17 Чтв 16:15:14 #339 №339670 
>>339668
Чисто углеродных планет не бывает. Так же как не существует сферических коней в вакууме. Но гипотетически да, нейтронная дыра, как естественный ускоритель частиц может удалбывая поверхность планеты высокоэнергетическими частицами ломать атомы, разбивать связи, отрывать электроны, создавать изотопы и творить прочие непотребства. На что хватит энергии прилетаемых частиц. В общем все, как в искусственных, человеческих ускорителях.
Аноним 18/05/17 Чтв 16:22:48 #340 №339671 
>>339669
А это принципиально невозможно, или в будущем может быть когда-нибудь найдут возможность создать историографию рукава галактики со всеми взрывами, движениями звезд и миграциями облаков газа...
Аноним 18/05/17 Чтв 16:53:04 #341 №339672 
>>339671
А вот хз, как фарш прокрутить назад. Насколько это реально можно будет прикинуть по результатам сбора данных GAIA есть тред, я там писал пост об этом. В двух словах Gaia ищет сестёр Земли по химическому составу. Если они найдутся, то можно будет примкнуть место расположения звёздных яслей. Или нельзя.
Аноним 18/05/17 Чтв 17:02:16 #342 №339673 
Можно ли добывать из атмосферы Венеры газ каким-нибудь рукавом, который бы всасывал газ в баллоны и консервировал его? Вот типа есть космический лифт, а это будет космический хобот газосос. Или он будет тормозить станцию газозабора?

Потом из этого газа можно добывать топливо какое-нибудь? Ну в смысле, можно ли заебашить экономику вокруг Венеры, индустрию по производству серной кислоты какой-нибудь или экзотического топлива, которое потом солнечными парусами шипить куда подальше за бабло?
Аноним 18/05/17 Чтв 17:27:38 #343 №339676 
анон, почему астероиды рангом поменьше - от 300 до 3 метров диаметре - нельзя найти, пока они не подлетят к земле?
то есть, есть он не летит с противосолнечной стороны, не подсвечивает (не блестит и не отражает почти из-за того, что весь из себя железокарбидный и поэтому матовый), то его и не найти вообще?
Аноним 18/05/17 Чтв 17:48:36 #344 №339677 
>>339674
Отсюда вывод. Для короткоживущего человека столкновение Сатурна, Урана и Нептуна с Юпитером всё таки будет выглядеть как взрыв солнца небольшой сверхновой, хотя дело в оптической иллюзии, росте краш-мусора планет и игры солнечного света на нём, совсем не обязана такая куча быть Траппистом, чтобы гореть путеводной звездой.
В общем, это было бы ОЧЕНЬ зрелищно.
Пойду "Меланхолию" фон Триера пересмотрю
Аноним 18/05/17 Чтв 18:24:36 #345 №339678 
Аноны, а почему лазер в космосе называют НИТВЕРДЫМ? Ну да, рассеивается луч, но кто мешает делать зеркало такого качества, чтобы луч практически не рассеивался на расстояниях, скажем, в световые минуты? Или меняющее форму зеркало, чтобы фокусировать его в зависимости от расстояния? В космосе луч нечем рассеивать же, хуле оно нитверда?
Аноним 18/05/17 Чтв 18:30:15 #346 №339679 
>>339673
Там атмосфера из СО2, что ты с ним собрался делать, наркоман? Мы свой-то не знаем, куда девать.
Аноним 18/05/17 Чтв 18:32:59 #347 №339680 
>>339678
Уже был вопрос про рассеивание лазора у космаси. Рассеивается и ещё как, там достаточно частиц на единицу объёма, что бы все твои труды накрылись пиздой.
Аноним 18/05/17 Чтв 18:45:25 #348 №339681 
>>339679
А из диоксида кислород извлечь можно же? Плюс азот для удобрений сжижать. Углерод на фуллерены.
Аноним 18/05/17 Чтв 18:47:19 #349 №339682 
>>339680
Недостаточно. Атом на кубометр - это луч площадью в кв.метр и прошедший расстояние в 1Мм поймает всего миллион атомов.
Это мало.
Очень.
Пиздец как мало.
Аноним 18/05/17 Чтв 18:59:59 #350 №339684 
>>339682
>Недостаточно.
Ага, а лазерный луч боженька веером разгоняет, ага.
>Атом на кубометр
Ебать манямирок.
Аноним 18/05/17 Чтв 19:06:40 #351 №339685 
>>339681
>А из диоксида кислород извлечь можно же?
Конечно. Только энергии нужно дохуя. Не слишком выгодно получается. Да еще с венеры везти, опять же нахуя? Выделяй ЦО2 на Земле и расщепляй, зачем за ним куда-то лететь?
>Плюс азот для удобрений сжижать
Азота дома сжижай, его блядь до сраки, аж 78% в атмосфере. Лететь на Венеру за ее 3.5% - совсем ебанулся?
Аноним 18/05/17 Чтв 19:09:47 #352 №339686 
MnZNuRfeuDA.jpg
>>339681
А теперь скажи - чисто с точки зрения затрат для мегакорпорации, которая бы стала инвестировать в такое - нахуя оплачивать разработку гигантской ракеты, с йоба-машиной, способной сосать атмосферу Венеры, при этом содерэащей в себе человека-оператора, пищу, воздух и лекарства для человека, его отхоже-погожее место, и всё с учётом постоянных рейсов ракет за баночками воздуха на другую планету и т.д. и т.п., если получать указанные тобой газовые смеси в банках можно ПРЯМО НА ЗЕМЛЕ ИЗ ЗЕМНОЙ АТМОСФЕРЫ?!
Что-то мне подсказывает, что учёным тебе не быть.
Аноним 18/05/17 Чтв 19:09:48 #353 №339687 
14450238921400.jpg
Есть ли Ip МКС?
Сидят, сидели ли космонавты на двощах?
Можно ли это проверить?
Аноним 18/05/17 Чтв 19:19:42 #354 №339689 
>>339687
Сап, Двач!
Я Юрчихин, пруфов не будет.
Аноним 18/05/17 Чтв 19:25:21 #355 №339690 
>>339687
Да, у них есть интернеты.
Аноним 18/05/17 Чтв 19:27:01 #356 №339691 
>>339687
>Можно ли это проверить?
Нет, нельзя. Трафик идет через внутреннюю сеть НАСА. Так что хуй тебе.
Юрчихин
Аноним 18/05/17 Чтв 19:28:10 #357 №339692 
>>339685
>Только энергии нужно дохуя.
Так у Венеры Солнце светит ярче, лол.
>Да еще с венеры везти, опять же нахуя?
Че сложного, хватай попутный ветер и лети. Те кто ближе к Солнцу всегда будут в дамках, а те кто будет за снеговой линией будут сосать хуй. Это же очевидно.

>>339686
Во первых не нужно участие человека, его можно свести к минимуму.
Во вторых венерианские производства нужны не для Земли, а для космической экономики. То есть ты конечно можешь добывать вещества на Земле, но тебе придется их поднимать из гравитационного колодца Земли, что будет увеличивать их стоимость. То есть тут речь будет идти о добыче в космосе и для космического потребителя. Понятное дело сейчас никому это говно нахуй не нужно, но в будущем при появлении потребителя его можно будет кому-то толкнуть.
Грабить атмосферу Земли к тому же вряд ли мудрая идея, учитывая, что низкоорбитальное производство может наебнуться, да и не ясно как люди отнесутся. А пиздить вонючку с Венеры это благое дело. Если еще и производство солнечных панелей на орбите организовать, или еще какой более сложный товар толкать, то вообще будет заебато.
Аноним 18/05/17 Чтв 19:29:03 #358 №339693 
>>339687
https://m.geektimes.ru/post/273942/
Аноним 18/05/17 Чтв 19:39:00 #359 №339695 
>>339692
>Че сложного, хватай попутный ветер и лети.
А, ты просто сельский дурачок, так и сказал бы.
Аноним 18/05/17 Чтв 19:44:23 #360 №339698 
>>339695
че орешь? Можно пенделя поддать лазером, если тебя ускорение не устраивает.
Аноним 18/05/17 Чтв 19:54:10 #361 №339699 
PobWx5uyjA.jpg
>>339698
Съебал бы ты уже в /b/, ты даже для ТТВ слишком тупой.
Аноним 18/05/17 Чтв 19:57:52 #362 №339700 
>>339699
>>339695
Это тред тупых вопросов, за которые в других местах выдают путевку в биореактор. Ну сказал анон что-то не то, какая у тебя реакция?
>"Фи, селяк, да хули ты тупые вопросы и гипотезы в треде тупых вопросов задаешь?"
Ты по теме ответить можешь? Или выебнуться решил?
Аноним 18/05/17 Чтв 20:08:11 #363 №339702 
>>339676
Потому что основное время они тусклее 22-й величины или находятся визуально близко к Солнцу или же находятся визуально в Млечном пути. Туда обзоры неба не лезут
Аноним 18/05/17 Чтв 20:17:48 #364 №339704 
>>338037 (OP)
Почему не бывает звезд зеленого цвета?
Я читал обьяснения что мол нагретое тело излучает волны света во всех частотах, включая зеленый, но глаз отфильтровывает только красный, желтый и голубой. Простите, но вот же зелёное пламя https://www.youtube.com/watch?v=B-fC-lZUa4M И если плазма может излучать виимый зеленый свет, то почему не может быть зеленой звезды?
Аноним 18/05/17 Чтв 20:22:46 #365 №339705 
>>339700
Тебе по теме все ответили, теперь ты просто кривляешься, клоун ёбаный.
Аноним 18/05/17 Чтв 20:33:47 #366 №339706 
>>339704
>то почему не может быть зеленой звезды?
И че, в какой звезде у тебя борная кислота горит в метаноле?
Предъявите пожалуйста новый механизм излучения звезд, получите нобелевку.
Аноним 18/05/17 Чтв 20:35:12 #367 №339707 
>>339684
> Ага, а лазерный луч боженька веером разгоняет, ага.
То-то на Луняшке по отражателям расстояние измеряли без мощных лазоров, и это-то с атмосферой Земли
> Ебать манямирок.
Поясни, епта. Я тебе не про НЗО. Предположим, ты где-то около Плутона пиздишься. Ни солнечного ветра, ни пыли почти, нихуя.
Аноним 18/05/17 Чтв 21:20:13 #368 №339712 
>>339678
Дифракционный предел, чтобы получить бесконечно прямую диаграмму направленности лазора нужна бесконечно большая апертура. А пока апертура будет конечная, луч всегда получится рассеивающийся с ростом расстояния.
Аноним 18/05/17 Чтв 21:28:58 #369 №339713 
>>339704
Иди загугли "цвета каления". Или "чветовая температура"
Аноним 18/05/17 Чтв 22:09:42 #370 №339721 
>>339707
>Ни солнечного ветра, ни пыли почти, нихуя.
Ебать манямирок.

>Поясни, епта.
Поясняю, что даже в ебучем межгалактическом пространстве в кубическом дециметре пара-тройка десятков атомов ионизированного водорода найдется. А в Солнечной системе, даже в самой жопе это число в несколько раз выше, не говоря уже об элементарных частицах.
>по отражателям расстояние измеряли без мощных лазоров
Да ты охуел? 3 вата уже приличная мощность. Да и похуй на мощность: там больше важно было длительность импульса. Им нужно было расстояние измерять, а не слово ХУЙ выжигать на поверхности. И получали, кстати, пятно в три километра диаметром.
Ну а самое большое разочарование для тебя - это учебник по ебучей физике. Если его открыть, то там будет написано что-то про свет и про его волновую природу. И так нехорошо для твоего манямирка получается, что даже если бы существовал идеальный вакуум, то в нем самый распиздатый лазер имел бы расходящийся пучок, так как физику не наебешь.
Аноним 18/05/17 Чтв 22:17:41 #371 №339724 
167884662711204499886391190084314725875712n.jpg
1707582913797020953832982543187059518996480n.jpg
Так, вопрос родился исходя и текущего обсуждения. Вывозим на орбита МКС лазер. Например, рубиновый. И не один, а много. А теперь лазер берёт и стреляет. Например, в Луну. Много ли теплоты снимет работа лазера?
Это если контекст задачи лазера избавляться станцию от теплоты.
А если наоборот использовать МКС как лазерной оружие?
С высоты-то много ли она сможет взрывать? Какой мощности должен быть лазер, чтобы с МКС прожечь лист стали или прожечь насквозь бетонную плиту на поверхности Земли?
Аноним 18/05/17 Чтв 22:57:09 #372 №339727 
>>339673
> Можно ли добывать из атмосферы Венеры газ
да
> каким-нибудь рукавом, который бы всасывал
нет
> Потом из этого газа можно добывать топливо какое-нибудь?
в ионники в теории можно любую хуйню залить, но идея довольно омская
>>339724
> Много ли теплоты снимет работа лазера?
отрицательное количество
Аноним 18/05/17 Чтв 23:01:47 #373 №339729 
>>339727
Как так?
Я ведь статью читал, где учёные научили излучением лазеров охлаждать
Аноним 18/05/17 Чтв 23:07:09 #374 №339730 
>>339729
ты читал про довольно специфическую хуйню (которую перед этим охлаждают гелием)
лазор при работе греется
а хороший лазор - дохуя греется
Аноним 18/05/17 Чтв 23:09:27 #375 №339731 
>>339724
>Какой мощности должен быть лазер
Невероятной, потому как пучок лазера, пройдя 400 км станет неприлично широким, а энергии на эту площадь пучка будет меньше, чем при излучении.
Аноним 18/05/17 Чтв 23:50:40 #376 №339745 
>>339706
Я читал, что звёзды светятся за счёт того, что пережигают время. Эта теория действительно многое объясняет. И повышенную температуру короны Солнца и земли.
Аноним 19/05/17 Птн 00:11:05 #377 №339746 
>>339731
А сколько копий лазерного пучка должны перекрывать это пятно с высоты 400км, что бы был эффект? Если на высоте 400км поместить по МКС с лазером на расстоянии 1км друг от друга? А если через километр орбитируют не МКСы, а некие машины - генераторы-накопители, каждый из который оснащён десятками, сотнями, а то и тысячами лазеров.
По-моему, годно. Например, радиус Земли 6371км, радиус НОО таким образом 6771км. Вычисляем длину окружности орбиты 42543,45км. А теперь смотри, если мы например хотим получить эффект, аналогичный эффекту выстрела пушки корабля инопланетян в "Дне независимости" и уничтожить Белый Дом, нам понадобится единовременно выпустить пучёк/луч лазеров. Не всех, логично же, а только тех, что в зоне видимости. Например, 30 градусов окружности - а это 14181,15 погонных км длины - со стороны Белого Дома в день, когда БД встанет на одной линии, и запускаем лазеры всех станций по 15 градусов от точки положения БД в обоих направления движения орбиты. Если расположить по 1 лазеру на 1 станции из 14181, и каждая из них иничиирует луч на БД, то...
А если на каждой станции по 10? По 100? По 1000 лазеров? И все они единовременно, да ещё и лучи мощноэнергетические....
Слушайте, а ведь Звезду Смерти в принципе не так уж и сложно создать, только она должна быть кольцом, а не планетой.
Аноним 19/05/17 Птн 00:13:40 #378 №339747 
>>339745
Я читал, что твоя мамка живет за счет того, что бухая дает всем подряд. Это действительно многое объясняет. И твой низкий интеллект и всратость.
Аноним 19/05/17 Птн 00:15:41 #379 №339748 
>>339746
Катился бы ты в сайфай уже. Там тебя полюбят и за 1000 лазоров и за День независимости и за Звезду смерти.
Аноним 19/05/17 Птн 03:21:45 #380 №339759 
Тоже хочу фанатеть от спейса, но не знаю с чего начать. Посоветуйте книжку какую нибудь
Аноним 19/05/17 Птн 09:27:53 #381 №339761 
>>339760
>у Головачева
Ну это вообще зашквар в степени зашквар.
Аноним 19/05/17 Птн 11:27:49 #382 №339775 
Безымянный.jpg
Будет ли работать и эффективна подобная хуйня? Водород разгоняется магнитным полем прямиком в прямоточный реактор, затем еще разгоняется катушками и выбрасывается из сопла?
Аноним 19/05/17 Птн 11:42:03 #383 №339776 
>>339760
Нет, это была научная книга.
Аноним 19/05/17 Птн 11:50:37 #384 №339777 
>>339775
>Водород разгоняется магнитным полем
А с чего ты решил что он будет разгоняться магнитным полем?
Аноним 19/05/17 Птн 11:53:00 #385 №339778 
>>339777
Бля, и правда. А если убрать первую катушку, а после реактора разогревать до плазмы?
Аноним 19/05/17 Птн 12:04:30 #386 №339782 

>>339759
https://2ch.hk/spc/res/306847.html
Аноним 19/05/17 Птн 12:40:33 #387 №339783 
В гипотезе ударного сотворения луны, сказано что Земля столкнулась с Тейей. Но ведь это была не земля.
Почему эту протопланету упорно называют землёй и как на самом деле она называется?
Аноним 19/05/17 Птн 12:57:35 #388 №339784 
>>339783
Земля не проходила стадию протопланеты?
А как ее называть? Мурзиком?
Название прото-Земля мистера говноеда устроит?
Аноним 19/05/17 Птн 14:39:50 #389 №339788 
14202057272880.jpg
>>339784
Земля всегда неизменна появилась сразу. Я так и понял. Аминь.
Аноним 19/05/17 Птн 14:44:56 #390 №339790 
>>339775
А в чём профит нагревания водорода? Вроде при нагревании он расширяется, но не думаю что прям критично много.
Аноним 19/05/17 Птн 16:00:56 #391 №339798 
>>339784
>А как ее называть? Мурзиком?
Карасиком же, ну.
Аноним 19/05/17 Птн 22:59:43 #392 №339835 
>>339798
>Карасиком
Раз уж тут тред тупых вопросов, поясните, откуда этот мемасик взялся
Аноним 19/05/17 Птн 23:02:26 #393 №339836 
3z2.jpg
>>339835
Спейсач образовательный
Аноним 19/05/17 Птн 23:03:29 #394 №339837 
>>339836
Первая картинка в гугле по запросу "Карасик", между прочим
Аноним 20/05/17 Суб 10:44:18 #395 №339863 
>>339677
>Меланхолию
Кусок говна, а не фильм.
Аноним 20/05/17 Суб 11:22:43 #396 №339866 
>>339775
А катушки зачем? Откуда ты для них энергию брать будешь? А так ты изобрёл NERVA.
Аноним 20/05/17 Суб 16:40:46 #397 №339881 
14887141567230.png
>>339863
Аноним 20/05/17 Суб 16:43:22 #398 №339882 
Пейсач, а какой профит получают gps-спутники от геосинхронности своей орбиты? Ну то есть ГЛОНАСС вроде тоже, как я понял, синхронизован, но с более ебанутым периодом 8/17. Так какой профит именно в 1/2? И как вообще на такие высокие круговые орбиты выводятся КА? Сначала на эллипс, а потом уже в апогее доразгоняют-доруливают?
Аноним 20/05/17 Суб 16:45:28 #399 №339883 
Где-то читал такую манятеорию, что пульсары - это сверхсветовые движки инопланетных кораблей. Что думаете?
Аноним 20/05/17 Суб 17:32:37 #400 №339885 
041214orbits.jpg
>>339882
>какой профит
Спутник всё время в одном месте, раз навёл антенну и забыл.
>ГЛОНАСС вроде тоже, как я понял, синхронизован
Ни GPS ни ГЛОНАСС не геосинхронные же.
>Сначала на эллипс, а потом уже в апогее доразгоняют-доруливают?
Да, пикрил.
Аноним 20/05/17 Суб 17:33:56 #401 №339886 
>>339883
Думаем, что не надо читать всякую хуйню в интернете.
>это сверхсветовые движки инопланетных кораблей
Не очень-то они далеко улетели. Точнее совсем никуда не улетели.
Аноним 20/05/17 Суб 18:46:48 #402 №339891 
>>338669
По-моему ниоткуда он не выезжал. Выездом из страны, логически, является въезд в другую. Космос - это не страна.
Аноним 20/05/17 Суб 19:12:15 #403 №339893 
>>339891
>Выездом из страны, логически, является въезд в другую.
Интересная логика конечно, но мне кажется, что "выездом из страны" считается "выезд из страны" - ну типа когда ты покидаешь территорию страны и оказываешься за её государственными границами. Таким образом логично предположить, что если ты сейчас не находишься в стране, а до того ты в ней находился, значит перед тем, как оказаться вне территорий этой страны, ты из неё выехал (выплыл/вылетел/выкопался).
А воздушное пространство государства заканчивается вроде как на высоте 110 км, значит Олдрин из страны выезжал.
Аноним 20/05/17 Суб 19:13:15 #404 №339894 
>>339891
>Выездом из страны, логически, является въезд в другую.
У тебя ущербная логика.
Выездом из страны логически является покидание ее территории.
Аноним 21/05/17 Вск 01:56:04 #405 №339927 
>>339893
>>339894
Ну хорошо, покидание территории через официальный таможенный пункт, на земле или в аэропорту, но ведь и этого не было.
Аноним 21/05/17 Вск 01:57:40 #406 №339929 
Я не думаю, что космонавты других стран при полётах разводят эту идиотскую бюрократию.
Аноним 21/05/17 Вск 11:59:23 #407 №339952 
>>339928
>>339863
Вот только Меланхолиядаже близко не сайфай и неаучпоп. Это артхаус. Тут главное картинка и атмосфера. Вот исходя из этого и надо оценивать.
Аноним 21/05/17 Вск 18:49:54 #408 №339974 
>>339955
>Что было бы если каждое небесное тело будет иметь в той или иной степени очень много органических молекул различной сложности и типа? Обязательно ли это сделало бы все небесные тела населенные хотя бы прокариотней? (Ну не считая звёзд и прочих)
До сраки всякой органики. Особенно в кометах. Жизни нигде не видно.
>Что если заменить 20-50% (Включая водород) всего вещества во вселенной на какашечки органические молекулы? К чему бы это привело?
Очень мало звезд, очень много планет из органики, вместо того, что бы из железа и кремния. Хуйня какая-то.

>Что если заменить все вещество в галактике Андромеда на антиматерию?
Перманентно бы излучала и истачивалась в результате движения через межгалактический водород и пыль. А через 4 миллиарда лет знатный и продолжительный фейерверк случился бы в местной группе.

>Как поведёт себя феномен расширения вселенной если бы вместо нашей наблюдаемой вселенной диаметром 96 млрд световых лет (с учётом расширившегося пространства) и всех этих галактик был бы горизонт событий такого же диаметра одной большой до шизы перебольшой чёрной дыры?

Да никак. Мы и так сычуем в гигантском пузыре, и что за ним нам не известно, так как не видно.

Последнее не понял: какой-то сумбур и ахинея.
Аноним 21/05/17 Вск 19:09:20 #409 №339975 
Может ли белый карлик стать нейтронкой?
Аноним 21/05/17 Вск 20:05:18 #410 №339980 
>>339975
Да, если наберет массы с компаньона и ебанет сверхновой типа Ia
Аноним 21/05/17 Вск 20:31:47 #411 №339984 
Сап тред, дайте пожалуйста гайд по типам сверхновых, по какой схеме обозначаются буквенно-числовые названия объектов в космосе, и вообще инфу о обозначениях в астрофизике
Аноним 21/05/17 Вск 20:43:37 #412 №339985 
>>339984
>гайд по типам сверхновых
Их хуй да нихуя.

>по какой схеме обозначаются буквенно-числовые названия объектов в космосе
Нахуя тебе? Алсо, их до сраки всяких разных, зачем этим заниматься, когда есть Sinbad?
Аноним 21/05/17 Вск 20:44:43 #413 №339986 
>>339985
>Sinbad
Simbad
фикс
Аноним 21/05/17 Вск 21:22:46 #414 №339987 
Если вдруг на Марсе найдут следы жизни - какую-нибудь кость окаменевшую, как это повлияет на всё?
Аноним 21/05/17 Вск 21:59:30 #415 №339988 
>>339987
>как это повлияет навсё
Сильно повлияет на всё
Аноним 21/05/17 Вск 22:50:18 #416 №339993 
>>339988
Так уж прям сильно? Инопланетная жизнь, тем более давно вымершая, вполне вероятна, то, что на Марсе была вода миллиарды лет назад - доказано. Ну новые марсоходы, исследования.
Аноним 21/05/17 Вск 23:09:28 #417 №339994 
PicUnrelated419847150.jpg
>>339993
Ну смотри: если найдут следы жизни, я пойду на улицу, обмажусь говном и начну дрочить. Если количество таких ебанутых среди населения Земли составляет например 1%, мы получим почти 73 миллиона Пахомов и Вселенная схлопнется. Очевидно же.
Верующие побугуртят, но в очередной раз придумают, почему это не идёт вразрез с их охуительными книжками, любители заговоров вместо "НАСА скрывает следы жизни на Марсе" станут кукарекать "НАСА сфальцифицировала следы жизни на Марсе", НАСЕ дадут пару дохуялионов на моар роверов и будут активнее пропихивать пилотируемый полёт, в интернетиках напилят кучу несмешных мемасиков на тему жизни на Марсе, антисетипетух сколлапсирует в сингулярность. Через год всем будет похуй примерно как сейчас всем похуй на динозавров.
Аноним 21/05/17 Вск 23:32:46 #418 №339996 
>>339994
>Верующие побугуртят, но в очередной раз придумают
Ну это совсем уж веруны не понимающие сути религии.
Зато вопрос об угле наклона оси и продолжительности суток двух планет будет популярнее.
Аноним 21/05/17 Вск 23:35:38 #419 №339997 
>>339996
> Зато вопрос об угле наклона оси и продолжительности суток двух планет будет популярнее.
статистика нихуёвая по земной группе так-то, с учётом двух развёрнутых
Аноним 21/05/17 Вск 23:42:24 #420 №339998 
>>339997
Все-таки есть совпадение. Так что или это не случайно - 2% разницы. Или наоборот - планетам в зоне обитаемости свойственно вращаться за сутки.
Аноним 22/05/17 Пнд 08:43:35 #421 №340007 
>>339998
Выборка маловата, что бы делать какие-то выводы. Особенно учитывая тот факт, что Земля вращалась быстрее когда-то и жизнь на ней при этом уже была.
Аноним 22/05/17 Пнд 09:28:02 #422 №340009 
>>340008
Еп, что в Андромеде, что у нас газопылевых облаков до ебаной тети. Достаточно посмотреть на рукава, что бы понять насколько все будет интересно. Конечно, за 4 млрд лет запасы поистощатся, но не критично. Сверхновые бахают точечно, а облака будут находить фронтами, учитывая разряженность и выделяемую энергию при аннигиляции, облака будут сталкиваться, разгоняться в противоположные стороны излучением, снова въебываться друг в друга и так далее.
Аноним 22/05/17 Пнд 09:56:40 #423 №340010 
>>340009
> Конечно, за 4 млрд лет запасы поистощатся, но не критично.
Это у анти-Андромеды истощатся, а нормальный МП подпитывается межгалактическим газом, так что запасы должны остаться примерно на нынешнем уровне, как я понимаю.
Аноним 22/05/17 Пнд 10:20:01 #424 №340011 
>>340010
> подпитывается межгалактическим газом
наоборот
Аноним 22/05/17 Пнд 10:22:04 #425 №340012 
>>340011
С чего ты взял? Именно что подпитывается на как минимум достаточном уровне, чтобы поддерживать металличность новообразующихся звезд на примерно одинаковом уровне.
Аноним 22/05/17 Пнд 10:51:27 #426 №340013 
>>340012
Потому что межгалактический газ оказывается там путём исхождения его из галактик.
Аноним 22/05/17 Пнд 11:00:03 #427 №340015 
>>340013
А можно какие-то пруфы на такое заявление, потому что оно явно и откровенно противоречит современным космологическим представлениям начиная от Большого Взрыва?
Аноним 22/05/17 Пнд 11:35:05 #428 №340017 
>>340016
Оно-то, наверное, самый логичный вариант, но, НЯЗ, сейчас аккреция межзвездного газа является практически общепризнанной, есть даже оценки темпов для МП, точных цифр не помню, вроде в районе 1-2 МСолнца в год, т.е. порядка скорости звездообразования внашей галактике.

А так мало ли, может человек просто эфирщик. Или какой-то из вариантов стационарной вселенной (в которых я нихуя не разбираюсь) такое предполагает. Хочется оценить степень маргинальности теории.
Аноним 22/05/17 Пнд 11:39:01 #429 №340018 
>>340015
А с каких пор все заявления должны не противоречить теории большого взрыва? Она когда-то стала общепризнанной?
Аноним 22/05/17 Пнд 11:55:33 #430 №340019 
>>340018
> А с каких пор все заявления должны не противоречить теории большого взрыва?
Extraordinary claims require extraordinary evidence. Они могут противоречить, но в этом случае хочется каких-то объяснений хотя бы для того, чтобы понять, что имеется в виду. БВ и так все знают, по крайней мере в общих чертах, а вот что может скрываться под теорией об исторгании галактиками межгалактического газа - загадка.
> Она когда-то стала общепризнанной?
В научном сообществе - да. Вне него до сих пор куча людей верит, что Солнце вращается вокруг Земли, потому срать на них.
Аноним 22/05/17 Пнд 12:18:18 #431 №340021 
>>340018
>Она когда-то стала общепризнанной
Точнее, когда приобрела характер факта.
Аноним 22/05/17 Пнд 13:08:22 #432 №340030 
image.png
>>340015
Миль пардон, я не думал, что придется выкладывать пруфы на давно известные факты.
>потому что оно явно и откровенно противоречит современным космологическим представлениям начиная от Большого Взрыва?
Нет-нет-нет, не противоречит космологическим представлениям, противоречит твоим заблуждениям и только.

https://www.britannica.com/topic/intergalactic-medium

>Very hot gas that emits X rays at tens to hundreds of millions of kelvins does indeed reside in the spaces between galaxies in rich clusters, and the amount of this gas seems comparable to that contained in the visible stars of the galaxies; however, because rich clusters are fairly rare in the universe, the total amount of such gas is small compared to the total mass contained in the stars of all galaxies. Moreover, an emission line of iron can frequently be detected in the X-ray spectrum, indicating that the intracluster gas has undergone nuclear processing inside stars and is not of primordial origin.

https://inis.iaea.org/search/search.aspx?orig_q=RN:20054503
>It is established that to an overwhelming degree the intergalactic medium consists of interstellar gas swept by galactic wind from the galaxies of the cluster into intergalactic space.

Если и это не успокаивает, то в гугле полно ссылок на тему "Origin of the intergalactic gas".

Так что вот этот >>340016 благородный дон, не смотря, что тоже не знал этого факта, сделал в целом верные выводы.

>>340017
>А так мало ли, может человек просто эфирщик.
А вот без оскорблений, пожалуйста.


Аноним 22/05/17 Пнд 14:37:23 #433 №340034 
>>340030
> статья 30-летней давности
Читать я её полностью, конечно, не буду. Впрочем, пробежавшись наискосок, немного удивился наличию в межгалактическом газе железа (причем почему-то только железа), надо почитать чего-нибудь по этому поводу. За прошедшие годы взгляды, видимо, слегка изменились, можешь вот эту довольно свежую статью как раз на нашу тему посмотреть: https://arxiv.org/abs/1612.00461 , там даже прямые наблюдения приведены. Или https://arxiv.org/abs/1612.00740 , например.

Какие-то выбросы из галактик (возможно - из активных ядер), вполне вероятно, присутствуют, но, насколько я разбираюсь в теме (могу ошибаться, естественно), для современных галактик потерю массы обычно не рассматривают как значимый эволюционный фактор, в отличие от аккреции газа.
> А вот без оскорблений, пожалуйста.
Пардон.
Аноним 22/05/17 Пнд 15:27:50 #434 №340039 

>>340034
Удивительное дело, я не отрицал аккрецию на галактики из межзвездной среды. Я говорил о происхождении межзвездного газа.
Аноним 22/05/17 Пнд 15:28:22 #435 №340040 
>>340039
Межгалактического
Аноним 22/05/17 Пнд 15:32:01 #436 №340043 
F2QnibwZLT0.jpg
Берём обитаемую планету с земной массой и гравитацией, но только размером с Марс или меньше.
Что о такой планете можно сказать?
Как там климат?
Что с магнитным полем?
Геология, тектоника прочие подземные процессы?
Аноним 22/05/17 Пнд 15:34:08 #437 №340044 
>>340039
> происхождении
Но ведь бред же. И да, фраза из статьи 30-летней давности - не пруф, увы. См. все вменяемые теории образования галактик за последние полвека.
Аноним 22/05/17 Пнд 15:42:17 #438 №340046 
1396213780158758282.jpg
>>340043
>Берём обитаемую планету с земной массой и гравитацией, но только размером с Марс или меньше.
Аноним 22/05/17 Пнд 16:09:22 #439 №340047 
>>340017
>эфирщик
Правильно, первым делом в борьбе с неугодным фактом нужно сделать его сторонников шизиками и жидами. Всё по Геббельсу.
Аноним 22/05/17 Пнд 16:50:15 #440 №340054 
>>340047
Можно просто сжигать на кострах или топить в реке. Я лично за этот подход.
Или, например, уфоконтактёров можно ссылать навечно в болота. Палеоконтактёров замуровывать в пирамидах, чтобы они сфокусировав силу вызвали подмогу и т.д.
Аноним 22/05/17 Пнд 17:13:37 #441 №340056 
>>340034
аккреция газа в три раза меньше истечения
результирующая порядка 2 съёбывающих масс солнца
> для современных галактик потерю массы обычно не рассматривают как значимый эволюционный фактор
смотря какой участок эволюции рассматривать
изначально говорилось о завтрашних 4млдр лет, на котором истечение довольно значимый фактор
Аноним 22/05/17 Пнд 17:31:40 #442 №340059 
>>340044
Нет, не бред. Это общепризнанно. Аргумент в стиле ваши "пруфы не пруфы"- абсурдно. Апелляция к времени написания статьи так же не состоятельны, многие вещи открыты давным давно, нет нужды переоткрывать открытое.
Если настаиваете, извольте принести контраргумент в виде исследования, которое это опровергает. А не отмахиваться в том стиле, который вы выбрали. А то странно получается, я свою точку зрения отстаивают научными статьями, а вы пустословством.
Аноним 22/05/17 Пнд 18:55:34 #443 №340069 
>>340046
Что не так?
Добавляем в ядро больше металлов и получаем большую плотность, значит, тот же вес при меньшем объёме
Аноним 22/05/17 Пнд 19:00:25 #444 №340071 
>>340056
> аккреция газа в три раза меньше истечения
Это для МП? Просто я, с одной стороны, не очень в теме, с другой - всё, что знаю, говорит об обратном. Надо будет закопаться поглубже. Какие там вообще механизмы? Вот, например в https://arxiv.org/abs/0803.3835 и https://arxiv.org/abs/1112.6247 считали, что взрывы сверхновых недостаточно сильны, чтобы газ достигал четвертой космической. Вот тут симуляция карликовой галактики, где тоже после первых двух миллиардов лет потери массы устанавливаются в единицы процентов за миллиард лет: https://arxiv.org/abs/1703.06442 . Для более массивных галактик, по идее, это будет еще меньше. Что кроме активных ядер (которых ни в МП ни в Андромеде нету в приложении к нашему случаю) вообще может выбрасывать вещество с такой скоростью? Или просто полет через межгалактический газ ведёт к такому, но тогда почему потери больше чем аккреция, с учетом немаленькой массы самой галактики?
> изначально говорилось о завтрашних 4млдр лет, на котором истечение довольно значимый фактор
С этой стороны я не смотрел, да.
>>340059
Покормлю, только ты сначала определись, ты за коммунистов или за большевиков? В смысле, верно ли я понимаю, что ты утверждаешь, что, как минимум, большая часть межгалактического газа имеет галактические происхождение?
Аноним 22/05/17 Пнд 19:01:23 #445 №340073 
>>340069
и большую гравитацию
че такой тупенький?
Аноним 22/05/17 Пнд 19:02:28 #446 №340074 
>>340069
Бессердечный эр квадрат. Ускорение свободного падения на поверхности планеты с массой Земли, но радиусом в 2 раза меньшим будет в 4 раза больше.
Аноним 22/05/17 Пнд 19:25:54 #447 №340076 
>>340074
Ну допустим.
Но плочему вдруг БОЛЬШЕ? На Юпитере при всей его массе и радиусе, хочешь сказать жэ будет мизерное?
Тем более, Марс с 4жэ, думаю, вполне будет пригоден для жизни. Просто нам, человекам, будет не легко там освоиться сразу.
Аноним 22/05/17 Пнд 22:52:22 #448 №340104 
>>340076
При 4ж будет сильная эрозия - капли воды во время дождя будут смывать все, растительность, почву, возможны только лишайники какие-нибудь.
Аноним 22/05/17 Пнд 23:07:07 #449 №340111 
>>340074
>Бессердечный эр квадрат
Только если массы точечные. На расстоянии меньше пары диаметров твоего шарика работать не будет.
Аноним 22/05/17 Пнд 23:07:21 #450 №340112 
Вот собирают всякие станции весом в 500 тонн на орбите, из кусочечков малых собирают, яко горлица сбирает льняныя семена.
Почему бояре не сделают корабль марсоходящий из модулей, вместо огромадной елды единостартующей?
Аноним 23/05/17 Втр 00:58:01 #451 №340130 
>>340071
>что взрывы сверхновых недостаточно сильны
Про ram-pressure stripping не слышал? Ясно. Про существование галактик, у которых закончилось активное звездообразование, при относительно невысокой металличности тоже? Понятно.
>которых ни в МП ни в Андромеде нету в приложении к нашему случаю
Если сейчас нету, это не значит, что не было, или не будет. А то вот два пузыря в Млечном пути нашлось откуда-то, сверху и снизу.
>но тогда почему потери больше чем аккреция
Скорость большая. Как на велосипеде ехать и ссать вперед. Неизбежно будешь мокрым.
>Покормлю, только ты сначала определись
Может ты перестанешь торговаться? Вроде не на базаре и принесешь таки какие-нибудь аргументы, подкрепленные хоть чем-нибудь, по поводу образования межгалактического газа?
>что ты утверждаешь, что, как минимум, большая часть межгалактического газа имеет галактические происхождение?
Это не я утверждаю, это утверждает наука.
Аноним 23/05/17 Втр 00:58:45 #452 №340131 
heic1404b1920.jpg
df3fermibubbleartlabels.jpg
n4402450.jpg
figure1b.jpeg
>>340130
Пикчи отклеились
ПЕРЕКОТ Аноним 23/05/17 Втр 02:29:44 #453 №340136 
ПЕРЕКОТ

>>340135 (OP)
https://2ch.hk/spc/res/340135.html
Аноним 03/06/17 Суб 02:53:24 #454 №341468 
>>340111
Если массы - однородные шары равной (внутри олного шара) плотности - зависимость та же.
Аноним 03/06/17 Суб 19:43:00 #455 №341569 
представим, что порталы из одноимённой игры существуют. А теперь подумаем: Что будет, если их поместить в вакуумной камере расположили 2 портала строго параллельно друг другу и бросить туда под прямым углом тело. Станет ли оно сингулярностью, разогнавшись до скорости света? (обретя бесконечную массу). Нет атм. сопротивления и на тело дейчтыует постоянное g
Аноним 04/06/17 Вск 10:12:33 #456 №341777 
>>341569
> представим
не представим, пиздуй в сайфач
Аноним 13/06/17 Втр 04:07:47 #457 №343120 
>>340112
В чем вопрос? Ты собрался стартовать в 500 тонн с земляши или собрать 500 тонн на орбите марса?
comments powered by Disqus

Отзывы и предложения